You are on page 1of 43

2014 NBE ONELINER

1. In the latent period of HIV infection, patient


develops: 8. Which of the following antiseptic is inappro-
1. Flu-like symptoms & lymphadenopathy priate for skin preparation of the operative site?
2. Progressive increase in HIV-1 viral titres 1. Chlorhexidine 2. Povidone Iodine
3. Progressive fall in CD4 counts 3. Cetrimide 4. Hexachlorophane
4. Systemic immune deficiency
9. Definition of systemic inflammatory response
2. Which of the following statements is TRUE syndrome (SIRS) includes all, EXCEPT:
regarding occupationally acquired HIV infection in 1. Hyperthermia (> 38C) 2. Bradycardia (60/min)
health care workers? 3. Tachypnoea (> 20/min) 4. WBC count < 4000/min
1. Risk is greatest during the earliest and latest stages of
the disease 10. Following regarding Tropical chronic
2. Principal route is by skin perforation with a solid pancreatitis is false:
needle containing HIV infected blood 1. Affects alcoholic elderly from lower socioeconomic
3. Post exposure HIV-prophylaxis should be started after region esp. South India
confirming HIV status of the source 2. Associated with ingestion of cassava
4. Post-exposure HIV-prophylaxis is with' zidovudine 3. Imaging studies reveal nodular fibrotic pancreas
(250 mg bd) for 3 month with dilated ducts filled with stones
4. Surgery reserved for intractable pain
3. What is of utmost importance in treatment of
wound abscesses? 11. Regarding typhoid all are true, EXCEPT:
1. Broad-spectrum antibiotics 1. Caused by gram Negative bacillus
2. Adequate surgical decompression and curettage of 2. Diagnosis is by leucopenia, stool and blood cultures
abscess and positive Widal test
3. Primary closure of wound 3. Perforation of ulcer in 1st week carries poor
4. Use of absorbable suture prognosis than 2nd week
4. Surgical intervention done in case of complications
4. A 38 year old obese lady develops pain &
swelling in her right leg along with fever & chills. 12. For in order of minimize surgical site in the
On examination, there is calor, rubor, dolor over a operation theatre all are true, EXCEPT:
poorly localized area over her calf with no pus 1. First scrubs should be for 3 mins.
pointing. WBC count is 18000/cumm. Blood culture 2. Bacterial count to be kept below 10 CFU/cmm
is negative. Diagnose her condition: 3. Temp should below 19-22C and humidity between
1. Calf abscess 2. Lymphangitis 45-55%
3. Cellulitis 4. SSSI 4. Good prepping and maintaining distance of
unscrubbed staff of atleast 50 cms from sterile field
5. All of the following are characteristics of a
MAJOR wound infection, EXCEPT: 13. Thymus gland abscess is congenital syphilis
1. Discharge of infected serous fluid are called:
2. Secondary drainage procedure may be required 1. Politizer's abscess 2. Fouchier's abscess
3. May be associated with SIRS 3. Duboi's abscess 4. Mycotic abscess
4. Planned discharge to home may be delayed
14. Malignant pustule occurs in:
6. A 40 year old man is operated for an abdominal 1. Melanoma 2. Anthrax
surgery via a midline abdominal incision. When is 3. Carbuncle 4. Actinomycosis
he most likely to develop a wound abscess post-
operatively? 15. Following are true of eryiseplas, EXCEPT:
1. 2-3 days 2. 7-9 days 1. Streptococcal infection
3. 10-14 days 4. 14-21 days 2. Margins are raised
3. Commonly seen in temperate region
7. Use of all of the following significantly decreases 4. None of the above
airborne infection in operation theatre, EXCEPT:
1. Laminar air flow 16. Commonest cause of acute LN-adenitis in India
2. Minimizing the no. of individuals in the OT is:
3. Air conditioning 1. TB 2. Lymphoma
4. High-efficiency particulate filters 3. Staphylococcal skin infection 4. Bare foot walking
2014 NBE ONELINER
3. GH levels decreased
17. Tetanus is caused by: 4. Glucose tolerance decreased
1. CI. Tetani 2. CI. Welchi
3. CI. Edematiens 4. CI. Septicum 25. During nutritional assessment of surgical
patients status of muscle protein is indicated by
18. 65 Kg male with 60% burns in catabolism is which one of following parameters:
admitted. An individual of this state requires 40 1. Serum albumin 2. Triceps skin fold thickness
Kcal/kg/day and 2 gm of protein/day/kg. This young 3. Hb level 4. Mid arm circumference
man is given solutions having 20% glucose and 25%
protein. If 3000 ml/day is given, then: 26. The type of naevus which is most likely to
1. Patient gets insufficient protein undergo malignant change:
2. Inadequate carbohydrate intake 1. Intradermal naevus 2. Compound naevus
3. Both protein and carbohydrate are adequate 3. Blue naevus 4. Junctional naevus
4. Too much protein is infused
27. Regarding Neurofibromatosis, false is:
19. Metabolic complications during parenteral 1. Un encapsulated tumors of schwann cells
nutrition include all, EXCEPT: 2. Resected along with nerve fibres
1. Hyperglycemia 3. Association with axillary freckling and Lisch noduls
2. Hypoglycemia 4. Always benign tumor
3. Hyperchloremic metabolic acidosis
4. Hypokalemia 28. Spontaneous regression of malignant tumors is
seen with:
20. Best method of post operative pain relief is: 1. Neuroblastoma 2. Retinoblastoma
1. NSAID suppositories 2. Intramuscular morphine 3. Malignant melanoma 4. All of the above
3. Acupuncture 4. Epidural analgesia
29. About cong torticollis all of the true, EXCEPT:
21. Metabolic features of 1. Associated with Breech deliveries
1. Low plasma insulin levels 2. Untreated cases may lead to
2. Decreased hepatic gluconeogenesis plagiocephaly
3. Increased hepatic glycogenolysis 3. Associated pterigium coli/Klieppel feil
4. Increased lipolysis syndrome
4. Always required surgical correction
22. In Evidence Based surgery all are true,' EXCEPT:
1. Aims to provide practice of surgery on a logical and 30. Ephilis is:
scientific basis 1. Type of freckle 2. Dental Anomaly
2. Emphasis on preparing systemic review and keeping 3. Infection caused by T.palidum 4. Pregnancy tumor
them up-to-date them
3. Includes randomized control trials and studies 31. In malignant melanoma, the margin of excision
4. Level V evidence or grade of recommendation D is the for tumor size 2cms is :
best evidence 1. 1 cms 2. 0.5 cms
3. 2 cms 4. 5 cms
23. For consent to be valid, informed it requires all,
EXCEPT: 32. Where are you most likely to use a full thicknes
1. Patient must be competent, not coerced into and well graft?
informed of pros and cons of procedures and other 1. Scrotum 2. Back
modalities available 3. Scalp 4. Face
2. Can be taken by junior member of the surgeons team
3. Explained in easy, lucid, language understood by the 33. The best reconstruction after a segmental
patient mandibulectomy in a patient of lower alveolus
4. Incase of childrens/ mentally II patients, parents / malignancy:
guardians are allowed for consent of procedure 1. Skin grafting
2. Pectorals major myocutaneous flap
24. In a person who has fasted for 5 days all are seen, 3. Delto pectoral flap
EXCEPT: 4. Free fibula flap
1. Free fatty acid levels in plasma increased
2. Immune reactive insulin level deceased 34. A deltopectoral flap is a type of :
2014 NBE ONELINER
1. Free flap 2. Myocutaneous flap 1. Uraemia 2. Rejection
3. Fasciocutaneous flap 4. Fasciomyocutaneous flap 3. Malignancy 4. Infection

35. The blood supply for a transverse Rectus 44. Hyperacute graft rejection is caused by :
abdominis myocutaneous flap comes from: 1. B - lymphocytes 2. Macrophages
1. Internal mammary artery 3. Preformed Antibodies 4. T-Iymphocytes
2. Superficial epigastric artery
3. Deep inferior epigastric artery 45. MOA of Sirolimus is :
4. Superficial external pudendal artery 1. Blocks-IL-2 gene transcription
2. Blocks IL-2 receptor signal transduction
36. Ideal graft for leg injury with 10 x 10 cm. exposed 3. Depletion and Blockade of T-cells
bone : 4. Prevents Iymphocyte proliferation
1. Amniotic memb graft 2. Pedicle graft
3. Full thickness graft 4. Split thickness skin graft 46. In renal transplantation in a living donor kidney,
Renal artery is anastomosed to :
37. Following are true of cultured bilayer skin 1. Ext. Iliac Artery 2. Renal artery
equivalent, EXCEPT: 3. Int. Iliac artery 4. Aorta
1. More closely mimics normal anatomy
2. easily handled, can be sutured or meshed 47. NOTES is :
3. Does not need secondary procedure 1. Non obliterative trans esophageal surgery
4. Long shelf life 2. Natural orfice trans esophageal surgery
3. Natural orfice trans luminal endoscopic surgery
38. Regarding Hemangiomas following are true: 4. Neo occlusive trans arterial Endo surgery
1. Salman patch disappears after age one
2. Portwinestain present through life 48. Triangle of doom has following Boundaries,
3. Salman patch seen on forehead or over occiput EXCEPT:
4. All are correct 1. Vas deferens 2. Testicular vessels
3. Iliac vessels 4. Reflected peritoneal fold
39. Criteria for Brainstem death include the
following, EXCEPT: 49. Length of the laparoscopic Hand instruments
1. Absence of corneal reflexes used/regularly in adult surgeries is:
2. Absence of motor response 1. 18cms 2. 28 cms
3. Absence of spontaneous respiration after 3. 25 cms 4. 36 cms
preventilation with 100% O2 for atieast 5 minutes, pt is
connected from ventilator for 10minutes with PO2 > 60 50. Graft-versus-host disease has occurred with the
mm of Hg transplantation of which of the following?
4. Tests performed by two clinicians on two separate 1. Kidney 2. Lung
occasions with atleast one of them a consultant from the 3. Heart 4. Bone marrow
transplant team.
51. The following are true of congenital
40. Optimal storage time in hrs for liver is : lymphoedema, EXCEPT:
1. < 24 hrs 2. < 12 hrs 1. Lower limbs are affected more often
3. < 10 hrs 4. < 3hrs 2. Usually unilateral
3. Onset usually occurs before puberty
41. In creation of peritoneum, gas used in 4. Episodes of lymphangitis worsens the edema
Laparoscopy is:
1. Coz 2. N20 52. In the management of leg ulcers, which of the
3. Helium gas 4. All of the above following is not done?
1. Cleaning the ulcer under tap water
42. The intra abdominal pressure during 2. Treating the skin of leg with emulsifying ointment
laparoscopy should be set between: 3. Use of topical antibiotics during dressing
1. 5-8 mm Hg 2. 10 25 mm Hg 4. Use of topical steroids to treat allergic response
3. 20 25 mm Hg 4. 30 35 Hg
53. Mr. Menon is due for a 10 hr flight to London. He
43. Principal cause of death in renal transplant has undergone knee replacement surgery one month
patients is: ago. How best can he prevent DVT from taking place:
2014 NBE ONELINER
1. Taking sleeping tablets
2. Avoid alcohol 62. All the following are good prognosis features of
3. Walk in the aisle occasionally Hodgkins disease, EXCEPT:
4. Low-molecular weight heparin administered before 1. Hb>10gm
the flight 2. Abs. lymphocyte count < 600/111
3. WBC < 15000/ cmm
54. The commonest lymphangiographic finding in a 4. Age < 45 years
patient with lymphoedema praecox is:
1. Congenital hyperplasia of lymphatics 63. Most malignant form of NHL is:
2. Proximal obliteration 1. Diffuse large cell 2. Small cell lymphocytic
3. Distal obliteration 3. Follicular lymphoma 4. Large cell follicular
4. Dysfunctional lymphatics
64. Podoconiosis is :
55. All of the following statement regarding filariasis 1. Type of fungal infections of feet
are TRUE, EXCEPT: 2. Endemic elephantiasis
1. Commonest cause of lymphoedema world-wide 3. Type of occupational chest infections
2. Microfilariae enter the blood at night 4. Malignancy of the nail
3. Diethylcarbamazine destroys the parasites and
reverses the Iymphatic changes 65. Odema pitting on pressure and disappearing an
4. Wucheria bancrofti is responsible for 90% cases bed rest and elevations is:
1. Grade II 2. Grade III
56. Most accurate diagnostic technique in 3. Latent or subclinical 4. Grade I
lymphoedema is:
1. Lymphangiography 2. Isotope Iymphoscintigraphy 66. A young child was brought by her mother with
3. CT scan 4. MRI swelling in the lower posterior half of neck which
became prominent on crying. On examination the
57. Decongestive lymphoedema therapy includes all, margins more not well defined and fluctuation was
EXCEPT: positive. Compressibility was possible diagnosis in
1. Skin care this patient is:
2. Manual lymphatic drainage 1. Cystic Hygroma 2. Bronchial cyst
3. Multilayer lymphoedema bandaging 3. Solitary lymph cyst 4. Cold abscess
4. Diuretics
67. Which of the following is not a feature of severe
58. Commonest cause of chyluria is : limb ischaemia?
1. Tuberculosis 2. Filariasis 1. Pain in calf on walking
3. Ascariasis 4. Malignancy 2. Rest pain
3. Coldness, numbness and paraesthesia
59. Meige's disease is : 4. Ulceration and gangrene
1. Congenital lymph oedema
2. Ovarian tumor with pleural effusion and ascitis 68. Salim had a cut throat injury following a brawl.
3. Lymphaedema precox He developed air embolism. Which of the following is
4. Lymphagio sarcoma in chronic Iymphoedematous incorrect regarding treatment of this condition?
limb. 1. Placement of patient in Trendelenburg position
2. Oxygen administration
60. Alemtuzumab is a chemo therapeutic agent with 3. Left side placement of patient
MOA as: 4. Aspiration of left ventricle
1. Antibody against CD20 Ag
2. Antibody against CD 52 Ag 69. False about fat embolism is:
3. Tyrosine kinase receptor inhibitors 1. Fat is metabolic in origin
4. Farnesyl transferase inhibitors 2. Patient becomes comatose with small pupils
3. Retinal changes take place late after onset of disease
61. Severe unilateral lymph edema is: 4. Petechial hemorrhages often occur
1. > 20 % excess limb volume
2. > 40 % excess limb volume 70. In all of the following, sympathectomy is effective,
3. > 30 % excess limb volume EXCEPT one:
4. > 50 % excess limb volume 1. Intermittent claudication 2. Hyperhydrosis
2014 NBE ONELINER
3. Raynaud's disease 4. Causalgia 2. Within the duct in the floor of mouth
3. At the opening of the submandibular duct
71. An ABI of less than suggests arterial injury even 4. None of the above
in the presence of palpable pulses:
1. 0.7 2. 0.5 80. Organism not found in acute bacterial parotitis is:
3. 0.3 4. 0.9 1. Haemophilus influenza 2. Staphylococcus aureus
3. Streptococcus viridans 4. None of the above
72. Gold standard for diagnosis of aortic rupture is:
1. 20 ECHO 2. X-ray chest PA view 81. Which of the following is not used as a method for
3. Aor togram 4. Multi slice CT scan with contrast prevention of Frey's syndrome during
parotidectomy?
73. While doing BK amputation, most important 1. Sternomastoid muscle flap
technical consideration is : 2. Lateral thigh flap
1. Stump should be short 3. Temporalis fascial flap
2. Ant flap longer than post flap 4. Insertion of artificial membranes between skin & the
3. Fipula transected above the tibial parotid bed
4. Nerves ligated at the level of muscles
82. Rajmohan aged 70 years had a lesion in the
74. . Regarding Aortic dissection following is false: region of level II in the neck which was cystic.
1. Presents as tearing intrascapsular pain Incorrect statement regarding this condition is:
2. Diagnosed by echo or CT/MRI with contrast 1. This is cystic degeneration from a small undetected
3. Control of BP is must before any further Imaging or primary squamous carcinoma in tonsil or tongue base
intervention 2. Radiotherapy is required
4. Type A usually are best managed medically 3. Laser excision may be done
4. There is no need for neck dissection
75. Cimino fistula is a fistula created between:
1. Radial A and cephalic vein 83. New man and seabrock's Sx (surgery) is done for:
2. Ulnar A and vein 1. Cleft palate 2. Cleft lip
3. Subclavian A and vein 3. Repair of parotid fistula 4. Chronic Iymphoedema
4. Long Saphenous vein and femoral artery
84. Ca of buccal mucosa commonly drain in to
76. Commonest site for ectopic salivary gland following lymph nodes sites:
tumor is: 1. Submental 2. cervical
1. Tongue 2. Cheek 3. Submandibular 4. Supraclavicular
3. Palate 4. Neck
85. A patient of Ca of Rt lateral border of tongue with
77. One of the following is not a reliable indicator level III lymph nodes on left side of a size 5 cms
of malignant change in submandibular salivary staging is :
gland: 1. No 2. N1
1. Facial nerve weakness 3. N2 4. N3
2. Pain in submandibular region
3. Rapid enlargement of swelling 86. Earliest tumor of appear after birth is :
4. Cervical node enlargement 1. Stemomastoid tumor 2. Ranula
3. Cystic hygroma 4. Thyroglossal cyst
78. Warthin's tumor is :
1. Malignant neoplasm 87. True about carotid body tumour is :
2. Rapidly growing 1. Arises form pharyngeal wall
3. Gives a hot pertechnetate scan 2. FNAC is done for diagnosis
4. Cold pertechnetate scan 3. May metastasize
4. Young adults affected
79. Carol presents with an acute painful swelling
in the region of the submandibular gland, which is 88. Muco epidermoid Ca of parotid arise from:
precipitated by eating & resolves spontaneously 1. Secrertory cells 2. Myoepithelial cells
over a period of 1-2 hours after meals. This would 3. Excretory cells 4. Myofibrils
mean presence of stone in:
1. The hilum of the gland
2014 NBE ONELINER
89. Which of the following statements regarding 2. Commonly seen in the Cantonese
chronic hyperplastic Candidiasis is not true? 3. Epstein-Barr virus is the infective agent
1. Lesions are common at oral commissures 4. Salted fish consumption is an important cause
2. Nystatin & amphotericin eliminate the infection
3. Re-infection after treatment is a definite 97. True about glomus-jugulae tumor, EXCEPT:
problem 1. M.C. in male
4. Surgery is never required 2. Arise from non-chromaffin cells
3. Fluctuating tinnitus and conductive type
90. Which of the following lesions is associated with deafness is earliest symptoms
an increased incidence of oral malignancy? 4. CT scan salt and pepper appearance of involved bone
1. Oral Lichen planus
2. Oral submucous fibrosis 98. A patient with Ca tongue is found to have lower
3. Dyskeratosis congenital neck positive nodes. The Rx of choice for the lymph
4. Chronic hyperplastic candidiasis nodes is :
1. Radical neck dissection
91. Regarding palatal cancers, all are true, EXCEPT: 2. Tele radiotherapy
1. Commonly seen where reverse smoking is 3. Suprahyoid neck dissection
practiced 4. Lower cervical neck dissection
2. Most of the tumours are of minor salivary gland
origin 99. Which Ca has best prognosis:
3. They present as sessile swellings & 1. Ca. lip 2. Ca check
ulcerate late 3. CA tongue 4. Ca palate
4. Deep infiltration into the underlying bone is
very common 100. Treatment of stage T3 N1 of Ca. maxilla is :
1. Radiation only 2. Chemo + radiation
92. Investigation of choice for diagnosis of 3. Sx with radiation 4. Chemo Rx only
oropharyngeal cancer:
1. Computerized tomography 2. MRI 101. MEN II A includes:
3. Radiography 4. Radionuclide studies 1. Ganglioneuromas 2. Cutaneous Lichen Amyloids
3. Hypocalcemia 4. Mutation in chromosome 10
93. Reconstruction after low-level maxillectomy for
a-hard palate can be carried out by : 102. During bilateral adrenalectomy ,
1. Latissimus dorsi flap intraoperative dose of hydrocortisone should be
2. Vascularised iliac crest graft given after
3. Fibular flap 1. Opening the abdomen
4. All of the above 2. Ligation of left adrenal vein
3. Ligation of right adrenal vein
94. Resection of mandible due to gross tumour 4. Excision of both adrenal glands
invasion is followed by primary reconstruction by
which of the following methods? 103. Commonest cause of Cushing's syndrome is:
1. Using a vascularised bone 1. Bilateral adrenal hyperplasia
2. Free corticocancellous graft 2. Adrenal adenoma
3. Alloplastic system supplemented with 3. Adrenal carcinoma
cancellous bone mush 4. Steroids
4. All of the above
104. Most common cause of Addison's disease is :
95. Invasion of the edentulous mandible in 1. Tuberculosis 2. Metastatic carcinoma
carcinoma of the floor of the mouth is : 3. Autoimmune 4. Amyloidosis
1. By deficiencies in the cortical bone of the alveolar crest
2. Via the periodontal ligament 105. Which of following is preoperative preparation
3. Above the insertion of the mylohyoid muscle pheochromocytoma?
4. All of the above 1. Fluids 2. Phenoxybenzarmine
3. Nifedepine 4. propanolol
96. Which statement regarding nasopharyngeal
carcinoma is not true? 106. Regarding Adrenal Incidentalomas following is
1. Commonest variety is adenocarcinoma false:
2014 NBE ONELINER
1. Detected in 4% patients on Imaging studies and 3. Mucoid 4. Colloid
prevalence increases with age.
2. Majority of them are phaechromocytomas 116. All are TRUE about CA breast, EXCEPT:
3. Hormonal evaluation is required 1. Affected sibling is a risk factor
4. Smaller masses usually are followed up 2. Paget's disease of nipple is Intraductal type of CA
3. Common in aged nulliparous
107. Malignant phaeochromocytoma differ from 4. Increased incidence with prolonged breast feeding
benign by ai, EXCEPT:
1. Breached capsule 117. Peau d'orange is due to :
2. Vascular Invasions and mets 1. Arterial obstruction
3. Ki-67 positive cells in high numbers 2. Blockage of subdermal lymphatics
4. Increased adrenaline levels 3. Invasion of skin with malignant cells
4. Secondary infection
108. Following statements about
craniopharyionmias is true: 118. Malti, a 45 years female patient with a family hlo
1. Tumors are uniforming solid breast carcinomas, showed diffuse microcalcification
2. Usually malignant on mammography. Intraductal carcinoma is situ was
3. May cause compressingof optic tracts and visual seen on biopsy. Most appropriate management is :
symptoms 1. Quadrantectomy 2. Radical mastectomy
4. Children with these lesion develop acromegaly 3. Simple mastectomy 4. Chemotherapy

109. Opsomyoclonus is encountered as elf of 119. Breast conservation surgery is indicated is one
1. Memingioma 2. Neuroblastoma of the following conditions:
3. Von tripped lindou disease 4. Neurofibromato 1. T1 breast tumor 2. multicentric tumor
3. Extensive in situ cancer 4. T4b breast tumor
110. Which of the following is the mc type of pituitary
yadenome? 120. Cystosarcoma phylloides is treated by :
1. Thyrotropinoma 2. Gonadotropinoma 1. Simple mastectomy
3. Prolactinoma 4. Corticotropinoma 2. Radical mastectomy
3. Modified radical mastectomy
111. A blood stained discharge from the nipple 4. Antibiotic with conservative treatment
indicates:
1. Breast abscess 2. Fibroadenoma 121. All of the following are used for reconstruction
3. Duct papilloma 4. Fat necrosis of breast of breast, EXCEPT :
1. Transverse rectus abdominis myocutaneous flap
112. A Female Patient present with a hard , mobile 2. Latissimus dorsi myocutaneous flap
lump in her rt. Breast . Which investigation would be 3. Pectoralis major myocutaneous flap
most helpful in making the diagnosis ? 4. Transverses rectus abdominis free flap
1. FNAC 2. Needle biopsy
3. Excision Biopsy 4. Mammography 122. A 14 year old healthy girl of normal height and
weight for age, complains that her right breast has
113. On mammogram all of the following are the developed twice the size of her left breast since the
features of a malignant tumor, EXCEPT: onset of puberty at the age of 12. Both breasts have a
1. Spiculation 2. Microcalcification similar consistency on palpation with normal nipples
3. Macrocalcification 4. Irregular mass areolae. The most likely cause for these findings is :
1. Cystosarcoma phyllodes 2. Virginal hypertrophy
114. Which of the following stage of Breast Ca 3. Fibrocystic disease 4. Early state of carcinoma
corresponds with following feature ~ Breast mass of
6 x 3 cm. size with hard mobile ipsilateral axillary 123. Large breast is not seen in :
lymph node and ipsilateral supraclavicular lymph? 1. Filariasis 2. Giant fibroadenoma
1. T4 N2 Mo 2. T3N1 M1 3. Cystosarcoma phylloides 4. Schirrhous carcinoma
3. T4N1 M1 4. T3 N3 Mo
124. For pregnant women who are found to have
115. Increased incidence with prolonged breast breast cancer :
feeding Breast a which is multicentric and bilateral: 1. Carcinoma of the breastbehaves more aggressively in
1. Ductal 2. Lobular pregnant women owing to hormones stimulation
2014 NBE ONELINER
2. Breast conservation is inappropriate for third
trimester pregnancies 131. 58 years old male presented with symptoms &
3. Most will have hormonally sensitive tumors Signs of progressive neurological deficits altered
4. Administration of adjuvant chemotherapy is safe for sensorium and irritability. On inquiry there was a
the fetus during the second and third trimesters h/o trauma 3 weeks back. Most likely diagnosis is:
1. Acute subdural haematoma
125. True statements regarding Pagets disease of 2. Chronic subdural haematoma
the breast include that is : 3. Extradural Haematoma
1. Usually precedes development of pagets disease of 4. Brain tumor
bone
2. Presents with nipple areolar eczematous changes 132. In multiple metastatic tumors of the brain ,
3. Does not involves axillary lymph nodes because it is a which of the following techniques is the most
manifestation of intraductual carcinoma only plausible option for treatment ?
4. Accounts for 10to 15 percent of all newely diagnosed 1. Surgery 2. Stereotic radiosurgery
breast cancers 3. Radio therapy 4. Steriods

126. Which of the following electrolyte disturbances 133. After a recent office visit for evaluation of
are most common in the head injured patient? headaches accompanied by nausea and vomiting and
1. Sodium 2. Potassium progressive truncal ataxia , a 4 year old boy is
3. Chloride 4. Bicarbonate diagnosed with medulloblastoma. Which of the
following statements regarding these tumors is true?
127. After a vehicular accident, Mohan went for a CT 1. Medulloblastoma is one of the rarer primary brain
scan. He was diagnosed to have a burst temporal tumors in children
lobe. This would mean: 2. First line treatment should include surgical debulking
1. Chronic subdural haematoma with radiation therapy
2. Contusional intracerebral haematoma 3. Chemotherapy with carmustine is effective at inducing
3. Extradural haematoma remission of medulloblastomas
4. None of the above 4. Most of these tumors arise from the floor of the fourth
ventricle
128. An 18 year old man is admitted to the
emergency room following a motorcycle accident. He 134. A 45 year old woman with a long history of
is alert and fully oriented but witnesses to the headaches has a large olfactory groove mass on
accident report an interval of unresponsiveness computed tomography (CT) scan. (SELECT 1 TUMOR)
following the injury. Skull films disclose a fracture of 1. Ependymoma 2. Oligodendroglioma
the left temporal bone. Following x-ray the patient 3. Meningioma 4. Glioblastoma multiforme
suddenly loses consciousness and dilatation of the
left pupil is noted. This patient should be considered 135. A middle aged man presents with a long history
to have: of seizures and a calcified frontal lobe lesion.
1. A ruptured berry aneurysm (SELECT 1 TUMOR) :
2. Acute subdural hematoma 1. Ependymoma 2. Oligodendroglioma
3. Epidural hematoma 3. Meningioma 4. Glioblastoma multiforme
4. Intraabdominal hemorrhage
136. A 25 year old woman presents with worsening
129. The term post traumatic epilepsy refers to hearing loss and tinnitus. A computed tomography
seizures occurring: (CT) scan reveals tumor involvement of cranial nerve
1. Within moments of head injury VIII bilaterally. What is it likely to be ?
2. Within 7 days of head injury 1. Von Hippel-Landau disease
3. Within several weeks to months after head injury 2. Prolactin-secreting adenoma
4. Many years after head injury 3. Neurofibromatosis type II
4. Growth hormone-secreting adenoma
130. Which of the following is commonest source of
Extradural hemorrhage? 137. In raised ICP with disrupted blood-brain barrier
1. Middle meningeal artery , which of the following drugs should be used with
2. Subdural venous sinus caution?
3. Charcot's artery 1. Steroids 2. Mannitol
4. Middle cerebral artery 3. Barbiturates 4. Frusemide
2014 NBE ONELINER
4. Urine / serum creatinine
138. A 6 Year old boy is found to have pigmented
hamartomatous lesions of the iris and an optic 146. Regarding Hypothermia, following is true,
glioma upon fundoscopic examination. Your EXCEPT:
diagnosis is : 1. Risk of ventricular fibrillation increases at
1. Von Hippel-Landau disease temp < 28 C.
2. Prolactin-secreting adenoma 2. Cardiac arrest occurs at 20C
3. Neurofibromatosis type I 3. Diagnosis confirmed by Rectal temp of 35 C
4. Growth hormone-secreting adenoma 4. At temp < 30C, heart is responsive to

139. A 25 year old woman presents with complaints 147. defibrillation and inotropes
of significant weight gain, amenorrhea, and "purple Half life of platelets is :
lines" on her skin. Select one of the following: 1. 24 hrs 2. 48 hrs
1. Von Hippel-Landau disease 3. 5 days 4. 9 days
2. Prolactin-secreting adenoma
3. Neurofibromatosis type II 148. Mc symptom of hemolytic transfusion a
4. Growth hormone-secreting adenoma conscious patient is :
1. Pyrexia and rigors
140. Suprasellar calcification with polyuria seen in: 2. Hemoglobinuria and oliguria 3. Burning sensation and
1. Langerhan cell histocytosis pain in
2. Medulloblastoma 3. the limb
3. Pinealoma 4. Urticaria and pruritis
4. Craniopharyngioma
149. Following is the best parameter in the
141. Not true regarding Dandy Walker cyst: management of shock:
1. Cerebellar vermis Hypoplasia 1. Blood Pressure
2. Posterior fossa cyst 2. Pulse oximetry
3. Long tract signs (+) 3. Deficiency of effective of circulation
4. Arachnoid cyst 4. CVP

142. True about Berry Aneurysm through is 150. Manohar admitted for leiomyoma of the
following, EXCEPT: stomach was to undergo elective surgery. However,
1. Mc site of rupture is apex which cause SAH he was diagnosed with Christmas disease. He will
2. Wall contains smooth muscle fibroblasts require which of the following blood fractions?
3. 90% occurs in Ant port of circulation at branching 1. Platelet concentrate 2. Fresh frozen plasma
points 3. Cryoprecipitate 4. None of t
4. Occasionally associated with NF1, coarctation of aorta.
151. Ormond's disease is caused by all, EXCEPT:
143. Witzelschult syndrome is seen in : 1. Hodgkin's disease
1. Temporal lobe tumor 2. Parietal lobe tumor 2. Inflammatory bowel disease
3. Frontal lobe tumor 4. Head injury 3. Ca breast
4. Focal glomerulosclerosis
144. Secondary haemorrhage differs from reac-
tionary haemorrhage in that: 152. Which of the following is not an anatomical site
1. It is due to rolling of a ligature of narrowing where a ureteric stone is arrested?
2. It follows restlessness, coughing & vomiting which 1. Crossing the vas deferens
raise the venous pressure 2. Uretero pelvic junction
3. It occurs due to infection & sloughing of part of the wail 3. Crossing the iliac artery
of an artery 4. Before entering the bladder wall
4. It takes place within 24 hours
153. Strangury is due to :
145. Characteristic finding of perenal azotemia in a 1. Mid ureter 2. Urethra
post operative patient is : 3. Inflammed bladder 4. Stretching of renal capsule
1. Urine sodium of 28 meq/L
2. Urine chloride of 15 meq/L 154. True regarding ureteric stones is all,
3. Fractional excretion of sodium less than1 EXCEPT?
2014 NBE ONELINER
1. Urine is always infected 3. Chronic cystitis
2. Pain is referred to tip of penis in 4. Malignancy
intramural stones
3. Source is always the kidneys 163. To differentiate between stress
4. Expectant treatment may be useful incontinence and Detrusor instability
investigation done is :
155. The following statements about the YAG laser is 1. Cystourethroscopy
correct, EXCEPT? 2. Urodynamic study
1. It can even cut the wire of stone baskets 3. MCU
2. Its use for uric acid stones has caused deaths due to 4. Retrograde urethroscopy
generation of cyanide
3. It has a wavelength of 2100 nm 164. For treatment of the ectopia vesicae,
4. It is effective against the hardest urinary stones which of the following bone is divided to
reach the site :
156. Tumor staging and grading in early bladder 1. Pubic ramic 2. IIiac bone
cancer is usually done with following 3. Ischium bone 4. Symphysis
1. USG 2. CT Scan
3. Cystography 4. Transurethral resection 165. A patient Kailash presents with haematuria for
many days. On investigations he is found to have
157. A 10-mm calculus in the right lower ureter renal calculi, calcifications in the wall of urinary
associated with proximal hydroureteronephrosis in bladder and small contracted bladder. Most probable
45 year old best treated with : cause is :
1. Open ureterolithotmy 1. Schistosomiasis 2. Amyloidosis
2. Extracorporeal shockwave lithotripsy 3. Tuberculosis 4. Ca urinary bladder
3. Antegrade percutaneous access
4. Ureteroscopic retrieval 166. About ectopic vesicae, following is true, EXCEPT:
1. Ca bladder may occur
158. Squamous cell tumor of urinary bladder is most 2. Ventral curvature of penis
consistently associated with: 3. Incontinence of urine
1. Stone 2. Schistosomiasis 4. Visible uretero - vesicle efflux
3. Chr. Cystitis 4. Diabetes mellitus
167. Rahul developed a neuropathic bladder after
159. True about bladder stones is all, EXCEPT: an accident. The lesion is found above T10. He has
1. Rare in Indian children incomplete bladder emptying but a good capacity
2. Primary stones are rare bladder. He can be managed by :
3. Small stones can be removed per urethra 1. Condom drainage
4. Maximum stones are radioopaque 2. Clean intermittent
3. catheterisation
160. False about Balkan nephropathy is : 4. Endoscopic sphincterotomy
1. Squamous cell carcinoma 5. Bladder reconstruction with fitment of artificial
2. Involves upper urinary tract urinary sphincters
3. Associated with consumption of grains stored in damp
environment 168. Most common complication of ileal conduit as
4. Nephron sparing surgery is done over nephrectomy a method of permanent urinary diversion is:
1. Ureteroileal stricture
161. Tear-drop bladder is a feature of : 2. Stenosis at the percutaneous site
1. Tuberculosis 3. Urine infection
2. Hunner's ulcer 4. Reabsorption of urine
3. Perivescial hemorrhage with rupture
4. Perivesical hemorrhage without rupture 169. Komal complained of urine loss during
laughing & sneezing. Which of the following is
162. One of the following disease will show urinary FALSE regarding her condition?
bladder calcification radiologically which resemble 1. It is usually seen in those with H/O caesarian
fetal head in pelvis: section
1. Tuberculosis 2. History of difficult labour with use of forceps
2. Schistosomiasis 3. Seen with epispadias
2014 NBE ONELINER
4. Symptoms may change with menstrual cycle 3. Erythroplasia of Queyrat
4. Bowen disease
170. Which of the following drugs is used for non--
adjuvant chemotherapy for bladder causes? 179. Rehman 33 year male presented with carcinoma
1. BCG 2. Gemcitabine of penis. At presentation there were no nodes
3. Doxorubicin 4. Vinblastine palpable in the groin. However he developed nodes
in the right. side of groin 2 months after surgery.
171. Diagnosis of bladder diverticulum is best Further treatment should include:
made on : 1. Observation
1. Cystoscopy with partially distended Bladder 2. Antibiotics
2. Intravenous urography 3. Chemotherapy & radiation
3. Retrograde cystography 4. Unilateral ilioinguinal node dissection
4. Cystoscopy with fully distended Bladder
180. Malignant transformation occurs in ulcer after
172. Following are the most common sites of many years of which of the following STI in the penis?
Ca bladder: 1. Lymphogranuloma venereum
1. Lateral walls 2. Trigone 2. Granuloma inguinale
3. Both of the above 4. None of the above 3. Condylomata acuminata
4. None of the above
173. The following are known causes of UTI in
females, EXCEPT: 181. Androgen production from a cryptorchid testis
1. Urethral stricture at 16 years of age is :
2. Colonization of perineal skin by E.Coli 1. Reduced to half of normal output
3. Neurogenic bladder dysfunction 2. Not reduced at all
4. Pre menopausal status 3. Reduced by 30%
4. Reduced by 75%
174. Carcinoma in schistosomiasis of the
bladder commences from which of the 182. Typical of Buschke-Lowenstein tumor is all,
following bilharzial lesion ? EXCEPT:
1. Papilloma 2. Nodule 1. Treatment is surgical
3. Ulcer 4. Pseudo-tubercle 2. Locally destructive;
3. Spreads to lymph nodes>
175. characteristics of the urethral syndrome 4. It is a verrucous carcinoma
does not include:
1. Symptoms of UTI 183. The most common cause of priapsm in of the in
2. Negative urine culture recent years is :
3. Absent pus cells in urine 1. Leukemic infiltration of pains
4. Interstitial cystitis 2. Spinal cord trauma
3. Sickle cell disease
176. True phimosis consist of following, EXCEPT: 4. Intracavernous injectiontheraphy
1. Fissuring of the preputial skin
2. Balanitis xerotica obliterans 184. Balanoposthitis is associated with all of the
3. Scarring of the prepuce following, EXCEPT:
4. Physiological adhesions 1. Penile cancer 2. Psoriasis
3. Gonorrhoea 4. Lichen planus
177. Excision of fibrous plaque in Peyronie's disease
cannot be replaced by: 185. Chemotherapeutic drug used against penile
1. Dermal graft calJper are all, EXCEPT:
2. Vein graft 1. Etoposide 2. Cisplatin
3. Tunica vaginalis graft 3. Methotrexate 4. Bleomycin
4. Non absorbable sutures
186. Non-filarial elephantiasis of the scrotum is due
178. The condition which is a carcinoma-in-situ is all, to infection with:
EXCEPT: 1. HIV 2. Lymphogranuloma venereum
1. Pagets disease of the penis 3. Chancroid 4. Syphillis
2. Giant condylomata accuminata
2014 NBE ONELINER
187. The following statement is false about 195. Patients should be advised to continue their
peyronie's disease: normal contraceptive precautions after vasectomy
1. Patients present with complaints of painful erection for a period of :
2. The condition affects adolescent males 1. 4-8 weeks 2. 8-10 weeks
3. The condition can be associated with Dupuytren's 3. 12-16 weeks 4. 16-20 weeks
contracture of the tendons of the hand
4. Spontaneous regression occurs in 50% of the cases 196. A 32 year male was having tender pain in his
right scrotum ; He has been on AKT for pulmonary
188. A patient comes with stage III non- Kochs since 1 month . All are false regarding his
serninomatous testicular tumor treatment of choice clinical picture , EXCEPT :
is: 1. There is a lax secondary hydrocele in
1. Radiotherapy 2. Chemotherapy 80% of cases
3. Hormonal therapy 4. Surgery 3. Beading of vas is a characteristic feature
4. Seminal Vesicle is normal
189. A 12 year old boy Naman presents with acute 5. A cold abscess could form in early cases
onset right scrotal pain. The pain is not relieved on
elevation of the scrotum. The testis is enlarged and 197. Treatment of carcinoma-in-situ of testicular
tender. There is no history of trauma. Which of the cancer in 40/M diagnosed by biopsy is :
following is the most appropriate management? 1. Surgery 2. External beam radiation
1. Immediate exploration 3. Chemotherapy 4. Interstitial radiation
2. Antibiotics
3. Psychiatric evaluation 198. Vithal 301M was diagnosed as a clo high stage
4. Antibiotics and scrotal elevation seminoma after investigations & high inguinal
orchiectomy. One of the following drugs is not useful
190. A patient .presented with a hard swelling in his for chemotherapy post surgery :
right testis. All are true statements, EXCEPT: 1. Vinblastine 2. Cisplatin
1. Trans scrotal biopsy is needed 3. Dactinomycin 4. Taxol
2. Inguinal exploration is done
3. High inguinal exploration should be done 199. Most common site of primary malignancy to
4. Scrotal U/S is done metastasise to the testis is :
1. Melanoma 2. Prostate
191. Not true about carcinoma penis is : 3. Kidney 4. Lung
1. Erythroplasia of Queret is a precancerous condition
2. 40% of patients are under 40 years of age 200. Vishals semen analysis report showed an
3. Circumcision if done anytime before puberty provides ejaculate volume of 1.5ml & sperm Concentration of
100% protection against carcinoma penis 21 million / ml. He had Suffered from mumps in
4. More than 50% patient have inguinal enlargement childhood :
when they present 1. Oligospermia 2. Normospermia
3. Azoospermia 4. None of the above
192. The most common non-germ cell tumor of the
testis is : 201. Rajnikant, 60 year old male is a case of Ca-
1. Leydig cell tumor 2. Gonadoblastoma stomach. CECT Abdomen showed a mass measuring 4
3. Sertoli cell tumor 4. None of the above cm x 4 cm in antrum with involvement of celiac
nodes. The management would be:
193. Besides LDH which of the following tumor 1. Palliative radiotherapy
marks are elevated in seminoma? 2. Palliative chemotherapy
1. AFP 2. hCG 3. Radical subtotal gastrectomy
3. Both of the above 4. None of the above 4. Total gastrectomy

194. The most common bilateral tumor of the testis 202. A 17 year old boy underwent splenectomy and
is: cholecystectomy for hemolytic anemia. On third
1. Teratoma differentiated 2. Seminoma post of day he developed sudden onset vomiting
3. Yolk sac tumor 4. Malignant lymphoma and abdominal distension. The management
includes all of the following, EXCEPT :
1. Ryles tube insertion
2. Normal saline intravenously
2014 NBE ONELINER
3. Nil per oral 2. Cauterization of ulcers
4. Surgery to relieve the distention 3. Truncal vagotomy and pyloroplasty
4. Ligation of gastroduodenal artery
203. The true statement about gastric volvulus is :
1. Mesentero axial is commonest variety 211. Commonest cause of duodenal fistula is :
2. Associated with Bochdaleck hernia 1. As a complication of gastrectomy
3. Endoscopy is investigation of choice for diagnosis 2. An abscess connected with perforated duodenal ulcer
4. Transverse colon hemiates into thoracic cavity 3. Traumatic rupture of duodenum
4. As a complication of Right colectomy
204. Nilima, 50 year old female had an epigastric
lump and gastric outlet obstruction. Investigation 212. Which of the following should be avoided in
revealed lump arising from stomach and infiltrating acute upper gastrointestinal bleed ?
liver and pancreas with paraaortic lymph node 1. Intravenous vasopressin
enlargement. UGIE Biopsy revealed poorly 2. Intravenous blockers
differentiated. Adenocarcinoma. The best 3. Endoscopic Sclerotheraphy
management would be : 4. Ballon Tamponade
1. Palliative RT
2. Palliative CT 213. In post gastrectomy nutritional syndrome
3. Radical total gastrectomy following occur , EXCEPT :
4. Palliative gastrojejunostomy 1. Calcium deficiency 2. Steatorrhoea
3. Constipation 4. vit. B12 defficiency
205. Most common benign tumour of stomach is :
1. Leiomyoma 2. Adenomatous polyp 214. Most likely cause of loss of periodicity of
3. Epithelial polyp 4. Lymphoma symptoms and sense of epigastric bloating in a case
of duodenal ulcer is :
206. All of the following are indications for surgery in 1. Gastric outlet obstruction 2. Perforation
gastric lymphoma, EXCEPT: 3. Carcinoma 4. Pancreatitis
1. Bleeding
2. Perforation 215. In a case of hypertrophic pyloric stenosis, the
3. Residual disease following chemotherapy metabolic disturbance is :
4. Intractable pain 1. Respiratory alkalosis
2. Metabolic acidosis
207. Most common site for carcinoma stomach is : 3. Metabolic alkalosis with paradoxical aciduria
1. Fundus 2. Body 4. Metabolic alkalosis with alkaline urine
3. Antrum 4. Lesser curvature
216. A 25 year old office executive presents with
208. All the following are incorrect about CHPS, recurrent duodenal ulcer of 2.5 cm size. The
EXCEPT? procedure of choice would be :
1. Heller's myotomy is the required surgery 1. Truncal vagotomy
2. Bilious vomiting 2. Truncal vagotomy with antrectomy
3. Commoner in first female child 3. Highly selective vagotomy
4. Hypochloremic alkalosis 4. Laparoscopic vagotomy and gastrojejunostomy

209. Early dumping syndrome is characterized by 217. Treatment of choice for Duodenal atresia is :
all of the following, EXCEPT: 1. Gastroduodenostomy 2. Duodenoduodenostomy
1. Occurs in second hour after meal 3. Duodenojejunostomy 4. Gastrojejunostomy
2. Is aggravated by more food
3. Patient has raised hematocrit 218. Correct statement about operation for morbid
4. Seen more commonly with Bilroth II obesity is:
1. Bacterial overgrowth in the By passed segment is a
210. Mr. Sujit a 45 year old office executive, a known complication of jejunoileal bypass
case of Acid peptic disease presents with massive 2. Following gastric bypass the patient may be
upper G.I bleed. Endoscopy revealed a bleeding permitted to resume normal eating habits
duodenal ulcer. The best line of management would 3. Long term weight loss is not sustained after a
be : jejunoileal bypass
1. Embolisation
2014 NBE ONELINER
4. Ulceration in bypassed antrum has been a problem 3. Selective angiography is diagnostic
after Roux-en-y gastric bypass operation. 4. Associated with aortic regurgitation

219. Incorrect about primary duodenal diverticuli is : 227. Abdominal actinomycosis usually manifests
1. Occur on concave border after which surgery?
2. Occur in 1st part of duodenum 1. Hernia 2. Nephrectomy
3. Are usually single 3. Cholecystectomy 4. Appendicectomy
4. Are accidental finding
228. Most common complication of hyphoid is
220. Incorrect regarding mesentericoarterial 1. Bleeding 2. Perforation
syndrome is: 3. Obstruction 4. Paralytic ileus
1. Caused by compression of distended duodenum
2. Common in young females 229. The Waldeyer's fascia separates the rectum
3. Does not occur in obese patients from the:
4. Most common in 5th - 7th decades 1. Uterus 2. Lateral pelvic wall
3. Prostate 4. Sacrum
221. A post operative patient presents with duodenal
leak along with signs and symptoms of peritonitis. 230. The chemotherapeutic agent of choice for
The most appropriate management is : adjuvant treatment in rectal cancer in 60 year male
1. Laparotomy with tube duodenostomy is :
and feeding jejunostomy 1. 5 fluorouracil 2. Adriamycin
2. Peritoneal lavage with drains 3. Paclitaxel 4. Epirubicin
3. Laparotomy and reanastomosis
4. Jejunostomy tube, duodenostomy tube and parenteral 231. Treatment of choice in colonic pseudo-
nutrition. obstruction since 2 days in 40/m patient is :
1. Masterly inactivity
222. Ladd's Band courses from: 2. Colonoscopic decompression
1. Splenic flexure to ileocecal junction 3. Emergency colostomy
2. 2nd part duodenum to right paracolic gutter 4. Emergency colectomy
3. Caecum to sigmoid colon
4. OJ flexure to right sacroiliac joint 232. Incidence of synchronous case of Ca colon is
malignancy in :
223. Following blunt abdominal trauma, a 12 year 1. 5% 2. 15%
old girl presents with upper abdominal pain, nausea 3. 25% 4. 50%
and vomiting, An upper gastro intestinal series
reveals a total obstruction of the duodenum with 233. All are true regarding a caecal Volvulus, EXCEPT
coiled spring appearance in D2 - D3. The final 1. Barium enema is usually diagnostic
definitive management is : 2. Usually anticlock wise twist
1. Gastrojejunostomy 3. Caecostomy may be required
2. NGT suction and observation 4. May occur as part of volvulus Eonatorum
3. Feeding jejunostomy
4. Evacuation of hematoma 234. In acute diverticulitis of the colon, the
sigmoidoscopic finding is :
224. Wind sock deformity is characteristic of : 1. Mucosa is inflammed
1. Duodenal atresia 2. Rectal atresia 2. Minute diverticuli seen
3. Biliary atresia 4. Oesophageal atresia 3. Saw toothed appearance
4. Sigmoidoscope cannot be passed beyond 15 cm
225. Most common type of Oesophageal perforation
is : 235. The organism which is commonly isolated from
1. Traumatic perforation 2. Latrogenic perforation the urine in high imperforate anus which is
3. Boerhaves syndrome 4. Corrosive injury indlcatlve ' of fistula is :
1. E.coli 2. Streptococcus
226. All are false regarding angiodysplasia of 3. Proteus 4. Staphylococcus
colon, EXCEPT:
1. Usually found is 30-40 age group 236. Most common cause of lower GI bleed in India is:
2. Rectum is the commonest site 1. Cancer rectosigmoid 2. Benign tumour
2014 NBE ONELINER
3. Non specific ulcer 4. Hemorrhoids 246. The maximum length of the rectum which can be
examined by a finger proctoscopy is :
237. Painless lower GI bleed is seen in child with: 1. 5 cm 2. 10 cm
1. Meckel's diverticulum 2. Rectal polyp 3. 15 cm 4. 20 cm
3. Anal fissure 4. Acute Appendicitis
247. Strawberry lesion of the rectosigmoid is caused
238. In case of 60/M with Ca colon true is : by:
1. Lesion on Lt. Side of the colon presents with features of 1. Spirochaeta vincenti
anemia 2. Streptococcus
2. Mucinous carcinoma' has a good prognosis 3. Gonococcus
3. Duke's A stage should receive adjuvant chemotherapy 4. Lymphogranuloma inguinale
4. Solitary liver metastasis is not a contraindication for
surgery 248. All are parts of the anorectal ring, EXCEPT:
1. Puborectalis muscle
239. Massive colonic bleeding in a patient of 2. Superficial external sphincter
Diverticulosis is from : 3. Deep external sphincter
1. Coeliac artery 4. Highest part of internal sphincter
2. Superior mesentric artery
3. Gastro duodenal artey 249. The commonest location of an anal fissure is :
4. Interior mesenteric artery 1. Midline posterior 2. Midline anterior
3. Left lateral 4. Right lateral
240. Non true regarding solitary rectal ulcer :
1. 20% are multiple 250. All are true regarding anal canal cancers
2. Recurrent rectal prolapse is a Cause EXCEPT:
3. Involves Posterior wall 1. Usually squamous cancer
4. Manage by digital reposition 2. Lymphatic spread is to inguinal nodes
3. More prevalent in
241. A 3 year old child comes with a partial rectal
prolapse. Your treatment 251. Which is false about Insulinoma?
1. Masterly inactivity 1. Mostly benign tumour 2. Weight loss
2. Digital reposition 3. Hypoglycemic attacks 4. Usually solitary tumour
3. Submucous phenol injection
4. Surgical excision of prolapsed mucosa. 252. A patient with chronic pancreatitis gives "chain
of lakes" appearance in ERCP examination. Rx of
242. The minimum safe distal margin for a Ca rectum choice is :
is : 1. Total pancreatectomy
1. 1.5cm 2. 2 cm 2. Sphincteroplasty
3. 2.5 cm 4. 5 cm 3. Side to side pancreaticojejunostomy
4. Resecting tail of pancreas and performing
243. True regarding the internal sphincter of pancreaticojejunostomy
canal canal is:
1. Circular muscles 2. Striated muscles 253. Regarding pancreatic trauma, false statement is:
3. Voluntary muscle 4. Supplied by autonomic nerves 1. Hyperamylasemia may occur without pancreatic
injury
244. Ove ideal time after birth to do an 2. Abdominal X-Rays are often not helpful in diagnosis
'Invertogram' test is : 3. Commonest mode is penetrating trauma
1. Immediately at birth 2. 1 hour 4. ERCP is mandatory in all cases of abdominal trauma
3. 3 hours 4. 6 hours
254. Treatment of choice for symptomatic annular
245. Commonest cause of tubular inflammatory pancreas is :
stricture of the rectum is : 1. Resection of pancreas 2. Duodenoduodenostomy
1. Iatrogenic 3. Observation only 4. Gastrojejunostomy
2. Endometriosis
3. Lymphogranuloma inguinale 255. One week after splenectomy for blunt
4. Schistosomiasis abdominal trauma a 36 year old female complains of
upper abdominal pain and lower chest pain which is
2014 NBE ONELINER
exacerbated by deep breath. She is ambulatory, 4. Hepaticojejunostomy
anorectic and making satisfactory progress. On
examination temp is 38.2C, decreased breath sounds 264. Which of the following is not an indication for
at left base. Abdominal wound is healing well and has cholecystectomy?
no signs of peritonitis . Rectal examination is 1. 40 year old male with symptomatic cholelithiasis
negative TLC is 12500/mm3 with left shift . 2. 15 year old male with sickle cell anemia and gall stones
Abdominal X-Ray Shows non specific gas pattern . 3. 30 year old male with large gall bladder polyp
Serum Amylase is 150 somogyi units. Most likely 4. 25 year old with asymptomatic gallstones
diagnosis is :
1. Pulmonary embolism 2. Pancreatitis 265. Which of the following is not associated with
3. Subphrenic abcess 3. Subfascial wound cholangio carcinoma:
infection 1. Gall stones 2. Ulcerative colitis
3. Sclerosing cholangitis 4. Clonorchis sinensis
256. Most common tumor of pancreas is :
1. Insulinoma 2. Lipoma 266. A patient presents with CBD stone of 2.5 cm 1
3. Gastrinoma 4. Glucagonoma year after cholecystectomy. The treatment of choice
would be :
257. Preferred management of Pancreatic abcess in 1. Supraduodenal choledochotomy
young male is : 2. Transduodenal sphincterotomy
1. Cystojejunostomy 2. Needle aspiration 3. Endoscopic sphincterotomy with stone extraction
3. External drainage 4. Cystogastrostomy 4. Transduodenal Choledochojejunostomy

258. The treatment of choice for a mucocele of gall 267. Treatment of type I choledochal cyst in 2 year
bladder in young female is: old infant:
1. Antibiotics and observation 2. Cholecystectomy 1. Cholecystectomy
3. Aspiration of mucous 4. Cholecystostomy 2. Cholecystojejunostomy
3. Cystectomy
259. Which of the following types of pancreatitis has 4. Excision and reconstruction with Roux loop .
the best prognosis:
1. Alcoholic pancreatitis 268. Treatment for symptomatic retained CBD stones
2. Gall stone pancreatitis in 40 year old female with cholecystectomy done 6
3. Post operative pancreatitis months ago:
4. Idiopathic pancreatitis 1. Medical dissolution of stones
2. Conservative treatment with antibiotics
260. Incidence of gall stone is high in : 3. Immediate surgery
1. Partial hepatectomy 2. Ileal resection 4. Endoscopic sphincterotomy
3. Jejunal resection 4. Subtotal gastrectomy
269. Contraindications for laparoscopic
261. Which one of the following is not a premalignant cholecystectomy are all, EXCEPT:
condition for Ca gall bladder? 1. Cirrhosis
1. Typhoid carriers 2. Cholecystoenteric fistula 2. Prior upper abdominal surgery
3. Rbrcelain gall bladder 4. Acalculous cholecystitis 3. Suspected carcinoma gall bladder
4. Mucocle of gall bladder
262. Clinical features of choledochal cyst in child is :
1. Pain, Fever,Progressive Jaundice 270. After exploration of common bile duct, the T-
2. Pain , Lump, Intermittent Jaundice Tube is removed on which of the following days :
3. Pain , Lump,Progressive jaundice 1. 6 Postop day 2. 4 Postop day
4. Pain , Fever ,Intermittent Jaundice 3. 12 Postop day 4. 3 Postop Day

263. Chhaya a 28 year old lady developed bile 271. Which of the following is false about splenic
leakage on 51 post cholecystectomy day due to artery aneurysms, EXCEPT:
lateral CBD tear. No T tube was inserted at the time of 1. Always have to be operated as mortality is high after
surgery. The ideal treatment is : rupture
1. ERCP and stenting 2. Occasionally cause bruit in left hypochondrium
2. Primary repair after reopening abdomen 3. Commoner in males
3. Ultrasound guided insertion of drain 4. Calcified ring on left side of LV-1 on plain X-ray
2014 NBE ONELINER
healthy and takes feeds regularly. You would
272. Splenectomy is most useful in : advise the mother to wait till the child grows upto :
1. Thalassemia 1. 6 months 2. 1 year
2. Sickle cell anemia 3. 18 months 4. 2 years
3. Hereditary spherocytosis
4. Acquired autoimmune hemolytic anemia 281. Desmoid tumour occurs in :
1. Breast 2. Brain
273. A patient of hereditary spherocytosis 3. Abdominal 4. Bone
underwent splenectomy for the same. She was
asymptomatic for 2 years. After which she started 282. Divarication of recti above the level of
having recurrence of symptoms. USG does not show umbilicus is seen in :
any abnormal lesion in abdomen. You schedule her 1. Babies 2. Elderly women
for relaparotoglY. You expect to find the possible 3. Multiparae 4. Malnourished men
cause of her problem at all the following sites ,
EXCEPT : 283. Sister Joseph module at umbilicus is from:
1. Porta hepatic 2. Splenic Ligaments 1. Stomach 2. Ovary
3. Mesocolon 4. Tail of pancreas 3. Breast 4. All of the above

274. Insullinoma is most commonly located in which 284. A 35 year old lady presents to surgery OPD
part of the pancreas: with pain and bleeding at the umbilicus. She gives
1. Head 2. Body history of such episodes during her periods. The
3. Tail 4. Equally distributed most likely diagnosis would be :
1. Secondary carcinoma 2. Omphalitis
275. The ideal treatment of stenosis of sphincter of 3. Endometrioma 4. Raspberry tumour
Oddi is:
1. Transduodenal sphincteroplasty 285. True statement about femoral hernia are all,
2. Endoscopic sphincterotomy EXCEPT:
3. Choledochojejunostomy 1. Occurs exclusively in females
4. Choledochoduodenostomy 2. Pregnancy is common cause
3. Strangulates
276. All the following can be used for treating 4. In males it is associated with cryptorchidism
femoral hernia, EXCEPT:
1. Lockwood operation 2. Lotheisen operation 286. Rasberry tumor is :
3. Moloney's operation 4. Stopas operation 1. Neoplastic 2. Inflammatory
3. Traumatic 4. Congential
277. All contribute to spread of peritonitis, EXCEPT:
1. Adulthood 2. Ingestion of food 287. Regarding to familial Mediterranean fever the
3. Steroid therapy 4. Use of enema true statement is , all , EXCEPT:
1. It manifest as a periodic disease with complete
278. Most common age of presentation of patent remissions in between
urachus is : 2. Commoner in females
1. Birth 2. Infancy 3. Peritoneum around gall bladder and spleen is inflamed
3. Adolescence 4. Old age 4. Colchicine is the causative agent

279. About Bochdaleks hernia all of the following are 288. Prevescial hernia is :
true, EXCEPT: 1. A type of direct hernia 2. A type of indirect hernia
1. In early presentation, the prognosis is better 3. A type of femoral hernia 4. Same as Spigelian hernia
2. Ultrasound gives early antenatal diagnosis
3. Commonly contains colon and stomach 289. Progressive bacterial synergistic gangrene is
4. Left sided is commoner than the right usually seen after:
1. Colostomy for perforated colon
280. A 2 month old male child is brought to your 2. Cholecystectomy
OPD. His mother has noticed a swelling near his 3. Colonic surgery
umbilicus which appears on crying and disappears 4. Laparotomy for a perforated viscus
when the child is quiet. The child otherwise is
2014 NBE ONELINER
290. The most common presentation of chronic
tuberculous peritonitis is: 300. Incorrect about obturator hernia is :
1. Loss of weight 2. Fever 1. Commoner in males
3. Pain abdomen 4. Ascites 2. Commoner in people > 60 year of age
3. PV/PR examination may
291. Exomphalos minor is a disease involving: reveal a tender swelling
1. Diaphragm 2. Cervix 4. Is usually Richter's type of hernia
3. Abdominal wall 4. Urinary bladder

292 .The pathognomonic sign of impending burst 1. (3)


abdomen is: Following infection by the HIV-1 virus into the blood, there is a
1. Shock brief seroconversjon illness that is characterized by flu-like
2. Pain illness & lymphadenopathy. There then follows a latent period
3. Serous sanguineous discharge when the infected subject appears well but has a progressive
4. Signs of intestinal obstruction fall in CD4 counts. In this phase, there is actually a fall in viable
HIV virus in the body that subsequently rises during
development of AIDS. Systemic immune deficiency occurs
293. Treatment for pneumoperitoneum due to
during. AIDS phase of illness.
perforation during colonoscopy is :
1. Temporary colostomy 2. Closure & lavage 2. (1)
3. Symptomatic 4. Permanent colostomy Risk with solid-needle is 10-fold less than with hollow-needle.
Post-exposure HIV prophylaxis should be offered when the
294. Most common organism causing omphalitis is: source patient from high-risk group & his HIV status is
1. Staphylococcus 2. Streptococcus unknown. As it should be started within 1 hour when possible
3. E-coli 4. Clostridium tetani it is inappropriate to wait to know HIV status. 3 drug
prophylaxis (Zidovudine, Lamivudine & indinavir) is given for
295. A 45 year old male develops pain in right lower 1 month.
abdomen while playing squash. The pain progresses
3. (2)
and she presents to the emergency room later that It is the most important aspect of treating wound abscess,
day with low grade fever, while blood cell count of whether antibiotics are used or not. Antibiotics usage is
20,000 and complaints of anorexia, nausea and controversial unless cellulitis, lymphangitis or related sepsis is
persistent sharp pain in right lower quadrant. On suspected. (3) & (4) are associated with poor healing
examination he is tender, in right lower quadrant, of wound abscess.
with muscular spasm and there is a feel of mass
effect. On ultrasound a mass in the abdominal wall is 4. (3)
seen. The most likely diagnosis is : Cellulitis is a poorly localized, non-suppurative invasive
1. Torsion of an ovarian cyst infection of tissues. All cardinal signs of inflammation are
present. Blood cultures are often negative, but SIRS is common.
2. Haematoma of rectus sheath
Lymphangitis presents with painful red streaks & painful
3. Acute appendicitis enlarged lymph nodes.
4. Strangulated Spighelian hernia Abscess is well-localised.
SSSI is superficial surgical site infection or an infected wound.
296. Colpotomy is done to treat:
1. Ischiorectal abscess 2. Pelvic abscess 5. (1 )
3. Appendicular abscess 4. Perianal abscess A major wound infection is defined as a wound that either
discharges significant quantities of pus spontaneously or needs
297. Most common cause for development of a secondary procedure to drain it (3) & (4) may also be
incisional hernia is : associated.
Minor wound infections will also discharge pus or infected
1. Obesity 2. Post operative cough
serous fluid, but is not associated with (2), (3) and (4).
3. Prolonged ileus 4. Post operative wound infection
6. (2)
298. Abdominal dehiscence is common on: Most abscesses take 7-9 days to form after surgery. As many as
1. 2nd post operative day 2. 4th post operative day 75% of infections may present after the patient has
3. 6th post operative day 4. 12th post operative day left hospital & thus, may be overlooked by the surgical team.

299. Lytles method is used for preparing: 7. (3)


1. Indirect inguinal hernia 2. Direct Inguinal hernia Control of air quality in a modern OT is important because
3. Femoral hernia 4. Umbilical hernia non-visceral bacterial contamination of the wounds is
predominantly from the air in OT. (1), (2) and (4) are all
2014 NBE ONELINER
effective methods to reduce post-operative air-borne infection. Commonest cause of acute LNadenitis is staphylococcal skin
Ultra-clean air in OT should have a bacterial count of less than infection and chronic LNadenitis is TB.
10 CFUs per cubic meter.
17. (1 )
8. (3) Tetanus is caused by CI. tetani.
One application of any alcoholic antiseptic is adequate for skin Gas gangrene is caused by CI. Welchi / oedematieus/ septicuss.
preparation & reduces the bacterial count by more than 95%.
Cetrimide (Savlon) has an aqueous presentation &. is useful in 18. (4)
hand-washing & instrument & surface cleaning. The patient is 65 kg and requirement of protein is 2
gm/kg/day that is 130 gms/day
9. (2) 25% protein means 25 gm per 100 ml. solution.
SIRS includes any 2 of following (1) Hyperthermia (> 38C) or That is 3000 ml solution contains 750 gm of protein.
hypothermia < 36C. Hence excess of protein is being transfused to this patient.
Tachycardia> 90. min or tachypnoea> 20 min.
WBC < 4000 or> 12,000/cmm 19. (2)
Sepsis is SIRS with documented infection Hypoglycemia typically occurs during cessation of TPN and
Sepsis syndrome is sepsis with incidence of one or more organ results from raised plasma insulin levels which lag behind.
failure like ARDSI renal failurel coagulation falling plasma glucose concentrations. This can be prevented
abnormality. by gradual reduction in the rate of TPN, rather that abrupt
cessation. (1), (3), (4) may occur during TPN.
10. (1)
It affects younger age groups and alcohol does not playa part in 20. (4)
the etiology, Ass with ingestion of cassaral Tapoica esp in Epidural analgesia is approaching a gold standard in post
southern India due to inability to detoxify cyanogens as a operative pain relief. Its use is associated with marked
result of malnutrition. Diagnosed by CT/USG with finding of reduction in mortality. They can remain in situ for 4 - 5 days
nodular fibrotic pancreas with dilated stone filled ducts. and provide excellent analgesia with either low concentration
Management is mainly medical with surgical Rx for local anaesthetics or opiates like diamorphine.
intractable pain (side to side pancreatojejunostomy done).
21. (2)
11. (3) Metabolic features of starvation
Typhoid is gram negative bacillus infection affecting peyers Low plasma insulin concentrations
patches of small intestines leading to necrosis and ulcerations. High plasma glucagon concentrations
If untreated may lead to malena secondary to bleeding or Hepatic glycogenolysis.
perforate (usually 2nd 13rd weeks). Diagnosis can be done by Protein catabolism
Widal (2nd week)1 Blood or stool culture (1st week)1 urine Hepatic gluconeogenesis
culture (3rd week). Usual Rx is medical with metronidazole I Lipolysis: mobilization of fat stores.
cephalosporin's and gentamicin. Surgery is required in patients Adaptive ketogenesis: Spares protein
with perforative peritonitis. Patient in 1st week of ulceration if Fall in energy requirements (to - 15 kcal kg-1 per day)
perforates has better prognosis as in early phase patient is less
nutritionally compromised and body defence robust also 22. (4)
better prognosis due to shorter interval between diagnosis and Level - I : Best evidence based on metanalysis and multiple well
operation. designed controlled studies. Level II to IV - Evidence available
but not so strong
12. (1) Level V - Evidence least compeling and based on case reports
First scrub should be about 5 mins. Usually chlorhexidine used and clinical examples. "A" grade recommendation is Best
(acts> 4 hrs)fuan iodine (action < 4 hrs). Ideally 20 air evidence and "D" grade is weakest evidence.
charges/hr with a 5 mm pore size filter is a adequate for a
general surgical theatre. Cotton suits below cotton gowns 23. (2)
decrease bacterial count by 30% in air and by 47% if elastic Person ideally should be the surgeon who will carry out the
anklets used ovser the trousers. treatment and may not have enough understanding to counsel
the patient properly.
13. (3)
Dubois abcess are thymus gland abscess in congenital syphilis. 24. (3)
Conlis also causes sabretibia, Hutichinsons Metabolic features of starvation includes:
teeth, Moons molars, Cluttons joints. Mycoticabcess are due to (a) Low plasma insulin concentrations.
fungal infection. (b) High g!ucagons levels.
(c) Hepatic glycogenolysis.
14. (d) Hepatic gluconeogenesis.
(e) Lipolysis.
15. (f) Fall in energy requirement up to 15 Kcal/ Kg day.
(g) Adaptive ketogenesis.
16. (3)
2014 NBE ONELINER
25. (4) It will be inferior in function and cosmesis. A delto pectoral flap
Nutritional assessment includes is useful for external skin defects and not inner mucosal
Fat: Skin fold thickness triceps. Biceps / Ant abdo wall. defects.

26. (4) 34. (3)


90% of malignant melanomas arise from junctional naevi. They A deltopectoral flap has only skin and fascia hence it is a
are likely to undergo proliferation from time to time. fasciocutaneous flap. It is based on the 2nd/3rd perforator
branches of internal mammary artery and used for head &
27. (4) neck reconstructions.
Malignant transformation of neurofibromatosis occurs in 5 -
10%. It is also called von Recklinghausen's disease 35. (3)
It is MC Hereditary Neurocutaneous syndrome It derives the blood supply from the Deep inferior epigastric
2 forms NF-1-Lisch nodules /cafe all lait spots artery and hence is also called the DlEA flap.
NF-2- Bilat vestibular Schwannomas. It can be used as a pedicled or free flap for breast
Both Autosomal dominant reconstruction.
Neurofibromas differ from Schwanomas in that
They are unencapsulated benign neoplasms of Schwann cells 36. (2)
and fibro blasts Tumor involves nerve and hence nerve needs For a skin graft (Full thickness or split thickness or amniotic
to be sacrificed. membrane) to survive it must be revascularized by recipient
bed. Radiation damaged tissues and relatively avascular
28. (4) structure such as bone, tendon, cartilages are therefore poor
Neuroblastoma, Retinoblastoma/RCC & malignant melanomas recipient sites.
have shown spontaneous regression in few cases. So an exposed bone surface is covered by a graft which has its
own blood supply. Such grafts are k/a flaps or pedicle grafts.
29. (4) Sabiston writes - "Pedicle graft or flap is a partially or
Congo torticollis is presents at birth causes are completely isolated segment of tissue with its own blood
Sternomastoid tumor (mc) supply"
Pterygium coli "Absolute indications for flaps-
Klippel feil syndrome. Exposed bone, radiated vessels, brain, an open joint or
Hemivertebrae nonbiological implant materials. Pressure sores where a bony
Usually associated with Breech deliveries leading to injury to prominence isexposed."
Sternocleidomastoid mastoid muscle
Rx includes -passive stretching, exercises and splinting 37. 4
Surgical release is used of consecutive Rx fails. Skin Advantage Disadvantage
substitutes
30. (1)
1. Cultured Off the shelf unstable
Epulis - Dental anomaly
allogenic No Biopsy Doesn't prevent
Syphilis Infection T.palidum
Keratinocytes needed wound contracture
Ephilis is a type of freckle
graft Provides wound Fragile
coverage Possible of disease
31. (3)
Provides healing transmission
If sqcellca < 2cms = 4mm margins required
Inadequate cosmesis
> 2 cms == 1 cm margins
2. Prevents Temporary
If malignant melanoma < 1 mm deep =1cm margin.
Bioengineered contracture dressings
Deeplesions = 2cm margin.
dermal Good re aration Decreased
These little data to support use of margin under than 2 cms
replacements for raft a lication reepithelisation
If BCC, margins b/w 2-15 mm depends on macroscopic variant
3. Cultured More closely Cost
Bilayer skin mimics normal Short half life
32. (4)
equivalent anatomy True en raftment
Full thickness grafts give a good color match and texture and
Does not need 2 questionable
hence good cosmesis. Only small areas can be grafted by full
procedure
thickness graft.
Easil handled,
The scrotum back and scalp do not require cosmetic grafts;
sutured or meshed
hence split thickness is better. Full thickness is
commonly used for the face.
38. (4)
33. (4) Capillary Hemangiomas are
A free fibula flap will give the best cosrnesis and function. 1) Salman patch - Present since birth
Skin graft is not an option as it will not give any tissue bulk and - Forehead or occiput
the contour is not even. A pectoralis major myocutaneous flap - Disappears by age 1 year
is an option if a free fibula is not possible. 2) Portwinestains - diffuse telengaotenia, no swelling
- Face/lips/Buccal mucosa presense since
Birth
2014 NBE ONELINER
- No tendency to disappear
3) Strawberry angioma - Bright red/dark red raised Hair 43. (4)
hemispherical swelling usually on Principle causes of death in Renal transplant patients in
head/neck & resembles a decreasing order is :
strawberry a) Heart disease
- Appears after birth grows till 1st b) Infection
year then resolves by 7-8th year. c) Stroke
4) Spidernomus - Usually acquired semina preg/t Infection is usually viral origin commonly CMV.
liver cirhosis.
44. (3)
39. (4) Types of Graft Reiection
Clinical testing for brainstem death Absence of cranial nerve Types Time Cause HP
reflexes Hyperacute Immediately Preformed Intravascular
Pupillary reflex antibodies thrombosis
Corneal reflex Acute 1st 6 T cell De en dent Mononuclear cell
Pharyngeal (gag) and tracheal (cough) reflex months immune infiltration
response
Occulovestibular (caloic) reflex
Chronic After 6 Non immune Myointimal
Absence of motor response months factors proliferation of
The absence of a motor response to painful stimuli applied to graft vessels
the head/face and the absence of a motor response within the
cranial nerve distribution to adequate stimulation of any 46. (3)
somatic area is an indicator of brainstem death. The transplant kidney is placed in the iliac fossa in
The presence of spinal reflexes does not preclude retroperitoneal position living the native kidney in situ. Donor
Absence of spontaneous respiration: Renal vein is anastomosed to Ext iliac vein. The donor renal A
After pre-ventilation with 100% oxygen for atleast 5 min, the is anastomosed end to side to ext iliac artery. If donor Arter:y
patient is disconnected from the ventilator for 10 min to lacks a Carel's patch like in living donor, it is anastomosed to
confirm absence of respiratory effort, during which time the intiliac artery end to end.
arterial Pco, level should be > 8 kPa (60 mmHg) to ensure
adequate respiratory stimulation. 47. (3)
To prevent hypoxia during the apnoeic period, oxygen (6 I NOTES is natural orifice transluminal endoscopic surgery. It's a
min") is delivered via endotracheal catheter. technique whereby the peritoneal cavity is entered
And test performed by 2 clinicians on two separate occasions endoscopically with natural orifices like month, anus or vagina
with atleast one is a consultant and none and surgery done ego Cholecystectomy and
associated with the transplant team. appendicectomy.
40. (2) 48. (3)
Max storage Optimal storage time Triangle of doom is bound by vas deferens, testicular vessels
Kidney 48
time (hrs) < 24
(hrs) and reflected part of peritoneal folds. It is a danger area of no
Liver 24 .< 12 tackers/sutures or dissection in lap Hemia Surgery. It contains
Pancreas 24 < 10
iliac vessels.
Small intestine 8 <4
Heart 6 <3
Lunq 8 <3 49. (4)
Variable length is 18-45 cms but regularly used instruments
41. (4) are 36 ems in adults and 28 cms for Paediatrics patients. 18
Gas Advantage Disadvantage cms to 25 ems for cervical cases and paediatric.
C02 Low risk of venous gas Hypercarbia and 45 cms = obese /very tall points.
embolus Does not support acidosis Vessels needle used is b/w 80- 120 mm in length.
combustion 80 mm - thin patients, 120 mm for obese patients.
N 2O Insignificant acid-base support combustion
change, Decreased pain in the presence of H2 50. (4)
or CH3 gas Donor-type lymphoid cells transplanted within a graft may
Helium Minimal effect on acid Risk of venous recognize the host's tissue as foreign and mount an immune
base balance embolism subacute response against the host. This response, termed graft versus
emphysema host disease (GVHD), is common in bone marrow
Argon Stableacid -base balance Cardiac depression transplantation and is an important source of morbidity and
mortality. Treatment requires more aggressive immuno
suppression. Current clinical practice includes depletion of
42. (2)
lymphocytes from the marrow graft in order to prevent the
In suffilator used to monitor the intra abdominal pressure and
development of GVHD, GVHD has been documented following
insufflate gas. Usually pressure set between 12-15 mm kg as
liver transplantation, presumably because of the large amount
high pressures cause hypercarbia/acidosis and adverse
of lymphoid tissue in the donor liver. GVHD has not been
cardiorespiratory problems.
2014 NBE ONELINER
described following heart, lung pancreas or kidney 59. (3)
transplantation. Aetioloaical classification of! Mohoedema
Primary Congenital (onset <2 Years old); sporadic;
lymphoedema familial (Nonne-Milroy's disease) Praecox
51. (2) (onset 2-35 Years old):
Congenital lymphedema is usually bilateral. It affects lower Sporadic; familial (Iatessier - Meige's
limbs more often and commonly occurs before puberty. disease)
Repeated lymphangitis causes obliteration of the already Tarda (onset after 35 years old)
deficient lymphatics and edema is worsened.
Secondary Parasitic infection(filariasis)
52. (3) lymphoedema Fugal infection (tinea pedis)
Topical antibiotics are ineffective in healing leg ulcers and are Exposure to foreign body material (silica
particularly likely to produce skin sensitization. They should particles)
never be used in the management of venous ulceration. Primary lymphatic malignancy
Patients who have eczematous reactions around their ulcers Metastatic spread to lymph nodes
may require the use of topical steroids to treat the allergic Surgical excision of lymph nodes
response. Trauma (particularly degloving injuries)
Superficial thrombophlebitis
53. (4) Deep venous thrombosis
The use of graduated compression stockings, exercise during Stewart Treve's syndrome is Iyrnphagiosarcoma in chronic
the flight and the avoidance of alcohol and sleeping tablets is lymphadernatous limb.
probably all that is required. In particularly high risk
passengers, that is those with a previous history of venous 60. (2)
thromboembolism, recent surgery or strong family history of Alemtuzumab Antibody agent against CD52 antigen - Rx of
thrombosis, low-molecular weight heparin can be lymphoma.
administered subcutaneously before the flight. Rituximab - Antibody against CD20 antigen - Rx of lymphoma.
Tyrosine kinases receptors inhibitors - Imatinib - Rx of GIST.
54. (3) Gefitinib/Erlotinib - Rx - Non small cell lung tumors
The commonest Iymphangiographic finding in lymphoedema Farnesyl transferase inhibitors = Lonafarnibl Tipifarnib -
praecox is distal obliteration with absent or reduced Against leukaemia
distal superficial lymphatics. It is also termed aplasia or
hypoplasia. 61. (2)
Severity of unilateral lymphoedema is classified as. Mild - <
55. (3) 20% excess volume
(1), (2) and (4) are true statements. DEC destroys the parasites Mod -20-40%
but does not reverse the lymphatic changes, Severe - > 40%
although there may be some regression over time.
62. (2)
56. (2) Bad prognosis in Hodgkin's disease seen with
Isotope Iympho-scintigraphy has now largely replaced - Albumin < 4 gm %
lymphangiography, which was previously considered the gold - Age> 45 yrs
standard. Radioactive Tc-labelled protein or colloid particles - Hb < 10.5 gm%
are injected into an interdigital web space and specifically - WBC 15,000/cm
taken up by lymphatics, and serial radiographs are taken with - Male
a gamma-camera. - Stage IV disease
- lymphocyte count < 600/l or 8% of WBC.
57. (4)
Management of primary lymphoedema required a holistic 63. (4)
approach. The current preferred term 'decongestive Working formulation for NHL is
lymphoedema therapy' (DL T) includes 2 phases. Low grade - Small cell lymphocyte
(1) 1st intensive phase (therapist -led) consisting of (1), (2), - Follicular small cleaved cues.
(3) and exercises. - Follicular small and large cleaved cells.
(2) 2nd maintenance phase (self -care) Int grade - Follicular large cell - Diffuse small cell
Diuretics are of no value in pure lymphoedema. - Diffuse large cell
- Diffuse mixed cell
58. (2) High grade - Lymphoblastic
Filariasis is the commonest cause of chyluria occurring in 1-2% - Immunoblastic
of cases 10-20 years after initial infestation. It presents as - Small non cleaved cells.
painless passage of milky white urine, particularly after a fatty Option a, c, d - come under Integrade. b is low grade. Of int
meal. It may also be caused by ascariasis, grade, most aggressive is diffuse large cell type.
Malaria, tumour and tuberculosis.
64. (2)
2014 NBE ONELINER
Endemic elephantiasis is also called as podocaniosis common Sympathectomy is of no value in intermittent claudication
in tropics. Where people walk bare foot in alkaline volcanic Possible indications for sympathectomy
rocks in Africa causing destruction of peripheral lymphatics by Ischemia disorders mainly of the limbs
silica particles. Plaster oedema develop is childhood and - Buerger's ds.
rapidly spreads proximally. Condition is prevented and - Raynaud's ds
progression slowed by wearing shoes. - Arterial injury or embolism
- Acrocyanosis
65. (4) - Erythrocyanosis
Grade Clinical features - Frost bite
(Brunner) Atherosclerotic ischemia of limb.
Subclinical There is excess interstitial fluid and Histological
(latent) abnormaiities in lymphatics and lymph nodes, 71. (4)
but no clinicall apparent lymphoedema An ABI of less than 0.9 suggest arterial injury even in the
I Oedema pits on pressure and swelling largely or presence of palpable pulses in patients with penetrating
completely disappears on elevation and bed rest injuries. This will prompt early arteriography and allow for a
II Oedema does not it and does not significantly successful early surgical intervention.
reduce upon elevation
III Oedema is associated with irreversible skin 72. (3)
changes that is fibrosis, papiIIae X-ray may show widened mediastinum but the gold standard
for diagnosis and of aortic rupture is the aortogram but a
66. (1) multi slice CT scan with contrast yields similar results.
Cystic hygroma is seen in young children / infants with
swelling increasing on crying with positive fluctuation, 73. (3)
transillumination and compressibility it is a lymphangioma; In BK amputation following considerations to be taken :
Excision of all sacs is the treatment. a) Nerves ligated higher up after pulling them down
Solitary lymph cyst - though congenital, Present is adult life. It b) Post flap longer than ant flap.
is a single cyst containing lymph develops in same c) Stump should be of adequate length not too short
process is cystic hygroma. d) Margins and Scar healthy central oranterior.
Branchial cyst - Cystic swelling due to persistence of cervical e) No dog earsl Bony spurts.
sinus although congenital, appears first around 20-25 year age, f) Fibula transected 2 cms above the tibial
lump is below angle of mandible. Fluctuation,
transillumination negative, Rx excision. 74. (4)
Aortic dissection
67. (1) - Most common affects ascending aorta
Ulceration and gangrene occur with severe arterial - man 50 - 70 yrs age, Afro- caribbeans.
insufficiency and often presents as a painful superficial erosion - Associated with HTN /marfans/preg/Co-arctation of aortic. -
between the toes. (3) is seen in moderate as well as severe Severe tearing intrascapular pain.
ischaemia and limbs become blanched on elevation and - Suspected on in X-ray chest seeing widened mediastinum.
develop a purple discolouration on dependency. - Traditionally aorto graphy is gold standard but can be picked
Rest pain in severe pain felt in the foot at rest made worse by up by 2D ECHO/CT/MRI also.
lying down or elevation of the foot. - 2 types A - involves ascending aorta.
Pain in calf on walking is seen in intermittent claudication. This B - does not involve ascending aorta.
is relieved by standing still. As the degree of ischemia increases Rx - Type A - Always surgery.
from I-III, the claudication distance decreases and it progresses Type B - medical Rx with control of Bp.
towards rest pain.
75. (1)
68. (4) Temporary Av fistula called cimino fistula noted at wrist
Placement in the Trendelenburg position encourages air to between radial A and cephalic vein.
pass into the veins of the lower half of the body. Placinq the If at ankle it is between post tibial A and near by vein.
patient on the left side will allow air to float into the apex of the
ventricle away from the pulmonary artery. 76. (3)
Oxygen is administered to counteract hypoxemia and to assist Palate is the most common site of ectopic/minor salivary gland
in the excretion of nitrogen. In serious cases, the right ventricle tumor. 80% of these tumor are malignant. Rx of these tumours
should be aspirated by a needle passed upwards and is surgical excision.
backwards from below the left costal margin.
77. (2)
69. (3) Pain is not a reliable indicator malignancy as benign tumours
One of the earliest signs may be emboli in the retinal arteries, often present with pain & aching in the affected gland,
which cause striate haemorrhages and 'fluffy' patches of presumably due to capsular distension or outflow obstruction.
exudates. (1), (2) and (4) are true. Besides (1), (3) & (4), induration and\or ulceration of
overlying skin is also a clinical feature of malignancy.
70. (1)
2014 NBE ONELINER
78. (3) N2a = Mets in single ipsilateral Ln size > 3 cms < 6 cms.
Warthin's tumour is a benign tumour arising most commonly N2b = multiple LN mets in ipsilateral none > 6 cms
from the parotid gland. It produces hot pertechnetate scan. N2C = mets in Bilateral or contralateral Ln none> 6 cms.
Prognosis is good. Treatment of this tumour is superficial N3 = Mets in LN > 6 cms size.
parotidectomy.
86. (3)
79. (3) 50-65% of cystic hygroma are present at birth they are due to
This classical picture occurs when the stone causes almost sequestration of a portion of jugular lymph sacs.
complete obstruction, usually at the opening of the Sternomastoid tumour - Injury to sterno mastoid muscle
submandibular duct. In (1) & (2) the symptoms are more in during delivery leading to fibrosis and contracture with
frequent, producing minimal discomfort & swelling, which is palpable mass palpated soon after birth.
not confined to mealtimes.
87. (3)
80. (2) - It is also called potato tumor or chemodectoma.
The most common organism to cause acute bacterial parotitis - Non chromaffin paraganglioma ass with pheochromocytoma.
is S. aureus. Streptococci, bacteroides and other - Arises from chemoreceptor cells from carotid Bulb.
anaerobe organisms are also implicated. - Usually unilateral.
- 10% cases have &.H/0 familial predisposition and affects
81. (2) people in 5th decade.
(1), (3) & (4) are all used as methods for prevention of Frey's - Slowly enlarging painless lump firm rubbery, pulsatile mobile
syndrome as they are local flaps. form side to side but not up down.
Frey's syndrome results from damage of the innervation of the - Confirmed by Doppler and CT angio. FNAC and biopsy are
salivary gland during dissection in parotidectomy surgery. contraindicated.
There is inappropriate regeneration of parasympathetic - Rarely metastasis.
autonomic nerve fibres which thus stimulate the sweat glands - Rx- Surgical excision If sized> 2 cms preop embolisation is
of the overlying skin. helpful.

82. (4) 88. (1)


This represents metastasis in a jugulo digastric node. This is It consists of - sq cells! mucus secreting of cells:
squamous cell carcinoma. (2), (3) & conventional excision are Inf. Hybrid cells and clear or Hydropic cells.
required. Recurrent disease is managed by surgery & Mixed parotid Ca caused by myoepithelial cells (Epithelial +
microvascular free flap reconstruction. Neck dissection is mesenchymal)
required in a large proportion of cases of advanced disease.
39. (4)
83. (3)
Rx for chronic lymphoedema includes charleslThompson and 89. (4)
Iympho venous anastomosis surgeries. Medical treatment with antifungal drugs may be necessary for
Sx for cleft lip is include millard's Rx, Lemesurier's Sx and many months to eliminate the organisms & reinfection is a
Tennessean's Sx. constant problem. Surgical excision is recommended for
Sx for cleft palate include Hynes and Wardills pharyngoplasty. persistent lesions.
Newman's' and SeaBrock's Sx is for repair and parotid fistula.
90. (2)
84. (3) Oral submucous fibrosis is not in itself premalignant but is
Lymph mets of buccal Ca goes to sub mandibular and upper associated with a higher than normal incidence of oral cancer.
deep central nodes. (1) & (3) are conditions about which there is doubt as to
Ca lip spreads to sub mental LNS; also form tip of tongue. whether their association with oral cancer is causal or casual.
Bilateral lymphatic spread is common in following tumors of (4) is a premalignant condition.
the head and neck
Lower lip Supraglottis Soft palate 91. (4)
In contrast with mandibular alveolar tumours, deep infiltration
85. (3) into the underlying bone is uncommon. Reverse smoking is
TNM staging for oropharyngeal Ca includes responsible for palatal cancers. Most of them arise from minor
T = No evidence of 1 tumor salivary gland.
T5 = Ca insitu
T1 = < 2 cms size 92. (2)
T2 = 2 - 4 cms size MRI is investigation of choice for oropharyngeal cancer. The
T3 = > 4 cms size advantage over CT is that the image is not degraded by the
T4 = involves adjoining structures! presence of metallic dental restorations. It is very good at
Muscles, Bones imaging soft tissue infiltration.
Nx = lymph nodes cannot be assessed Plain radiography is of limited value in investigation of oral
N0 = No LN mets cancer. At least 50% of the calcified component of bone must
N1 = Mets in single ipsilateral size <3 cms. be lost before any radiographic change is apparent.
2014 NBE ONELINER
Radionuclide studies are of limited value in diagnosis of and IV
primary oral cancers. So for same stage, Ca lip has highest 5 year survival rate or
have best prognosis.
93. (4)
Besides these three, radial forearm flap & rectus abdominus 100. (3)
flap can also be used for reconstruction. Maxillary Ca is two types:
(a) Sq cell Ca (m.c. type)
94. (4) (b) Adeno ca
All of the above are techniques followed for primary Rx of Ca maxilla is
reconstruction of the mandible. Sq cell ca -7 combination of Sx and RT gives better results than
either Rx alone. (Rx is same in every stage of maxillary Ca).
95. (1) Adena ca - only Sx done as RT is in effective.
Invasion in the edentulous mandible is almost exclusively via
deficiencies in the cortical bone of the alveolar crest. In the 101. (4)
dentate mandible, invasion is usually via the periodontal MEN - II syndromes are due to mutation in RET.
ligament & is nearly always above the insertion of the Protooncogene present on chromosome 10,
mylohyoid muscle. Once the tumour has invaded the mandible, MEN IIA - Medullary thyroid cancer, Pheochromocytoma
it soon enters the inferior dental canal & perineural spread hyperparathyroidism.
occurs anteriorly & posteriorly. MEN II B - Medullary thyroid cancer. Pheochromocytoma.
Mucosal neuromas, megacolon, skeletal abnormalities.
96. (1)
Commonest variety is undifferentiated squamous cell 102. (4)
carcinoma. Only few lessions are well differentiated squamous Excision of both adrenal glands
cell carcinoma & these two lesions make up over 90% of Schwartz
nasopharyngeal malignancy in endemic areas. "Patients undergoing surgical treatment of endogenous
Rest all are true. hypercortisolism require glucocorticoid replacement steroids
are not given preoperatively because these patients are
97. (1) already hypercortisolemic. Instead hydrocortisone 100 mg i.v,
- non chromaffin producing paragangliomas is given after the removal of the~cor}qhYperplastic adrenal
- middle aged females affected gland.
- Non encapsulated, extremely vascular and locally invasive C.S.D.T.
tumour but can metastizes to the lung/liver/ LNS "After total adrenalectomy, life long corticosteroid
- Usually arise from adventitia of jugular bulb (gl. Jugular) or maintenance theraphy becomes necessary. The following
Jackson's N in temporal bone (gl. tympanicum). schedule is commonly used. No cortisol is given until the
- Earliest symptoms is deafness (conductive with pulsatile adrenals are removed during surgery.
tinnitus) On the firest day 100 mg iv is given every 8 hours.
- Rising sum appearance on otoscopy. Love & Bailey
- Angiography provides a definitive diagnosis According to L&B, 23rd /e corticosteroid replacement is
- CT shows salt paper appearance of involved base. necessary during both intraoperative and postoperative while
- TOC is surgery with preop embolisation according to 24th/e corticosteroid therapy intraoperative is
only needed when the patient has been treated with
98. (1) Ketoconazole or mitotane.
Rx of HFN tumors when there is involvement of cervical lymph L&B 23rd /e
nodes, the 10 and nodes are both treated surgically. The gold "It is essential that all patients who are to be subjected to
standard has been radical neck dissection, all modification of adrenalectomy are supported intraoperatively and
neck dissections are described in relation to std radical neck postoperatively by adrenocortical hormone replacement
dissection. therapy irrespective of the extent of adrenal resection ,
Rx of Ca tongue size < 1cms = Sx removal with wide margins corticosteroids are started when anaesthesia is induced,"
large mass - RT is 10 Rx and Sx reserved for salvage. Growth L&B 24th
reaches with in 2 cms of jaw Sx is 1 mode with pre or post " In the new edition they have removed these lines and states
op RT as adjuvant Rx. that
Reconstruction required if defect. Only patients who have received medical therapy with
< 1/3rd tongue - Not required mitotane and Ketoconazole should be given
> 1/3rd < 2/3rd tongue - radial forearm flap Corticosteroid at the induction of anaesthesia and also adds
> 2/3rd tongue - pectoralis major flap. that "All patients who have undergone adrenal surgery
(U/L or B/L) will require Post operative steroid replacement.
99. (1)
5 yr. survival rates 103. (4)
Site Lip Tongue Palate Check All the given options can cause Cushing's syndrome. However
Stage I 90% 70% 80% 65-75% exogenous administration of steroids for treatment is most
and II common cause of Cushing's syndrome. Most common
Stage III 50% 40% 40% 30-50%
2014 NBE ONELINER
endogenous cause for Cushing's syndrome is pituitary At least 50% of children with opsoclonus tonus-myoclonus
dependant Cushing's disease. have an underlyiqgneuroblastoma.
Opsoclosus is a disorder of eye movement characterized by
104. (3) involvements chaotic saccades that occur in all irections of
Autoimmune disease is responsible for 60% of cases of gaze, frequently associated with myodonus and ataxia.
Addison's disease. Remaining options viz infections, Cause - 1. Idiopathic
amyloidosis and metastases constitute remaining 40%. 2. Cancer related (paraneoplastic
e.g. Neuroblastomas in children and lung and breast CA in
105. (3) adults.
Fluids are used to preload the patient for him to withstand
virtual hypotension caused by removal of tumour, propranolol 110. (3)
is used to initiate blockade, phenoxybenzamine is Prolactinomas - 52%
irreversible a blocker is first drug of choice for bringing down GH cell adenomas 27%
BP in these patients. Nifedepine has no role in preoperative Hormone Negative adenomas - 21%
preparation of these patients. Corticotropinomas - 20%
Thyrotropinomas 0.3%
106. (2)
Ad renal Incidentalomas are clinically unapparent masses 111. (3)
detected accidentally by imaging studies done for other reason. "Intraductal papilloma is the most common of bloody nipple
They are defected in 4% cafes and prevalence Increases with discharge."
age. "Bloody discharge is more suggestive of cancer but is usually
- More than 75% cases are non functional adenomas only 4% caused by a benign papilloma in the duct"
are Pheochromocytomas CSDT
- Complete Hist/Examn and biochemical work up is required Also remember
with CT/MRI. - Nipple discharge is suggestive of cancer if it is spontaneous,
- Hormonal evaluation includes 24hrs excretion of the unilateral, localized to a single duct, occurs in women age of 40
catecholamine, metanephrines, 1mg overnight dexamethasone years or more, is bloody, or is associated with a mass.
suppression test, SrK+, aldosterone & renin activity Sr DHEAS, - Nipple discharge is suggestive of a benign condition if it is
testosterone or 17--OH estradiol. bilateral or multiductal in origin, occurs in
- FNAC done after phaeochromocytoma is ruled out Women age 39 years or less, is milky or blue green in colour.
- Rx smaller non functional masses <4 cms -following up with
CT/Hormone analysis at 6, 12, 24 mths. Mass > 4cms / 112. (3)
functional mass are excised. An open biopsy is the most definitive investigation
- "The most reliable diagnostic test for breast cancer is open
107. (4) excisional biopsy. When the sample is properly taken by the
The difference between malignant and benign tumors is surgeon and examined by the pathologist, such a test should
difficult except if mets are present. An increased PASS give no falsenegative and no false positive results."
phaeochromocytoma mass of phaeochromocytoma of the - "Large-needle (core needle biopsy) is an accepted dlaqnostic
adrenal gland scale score indicates malignancy as does a high technique n which a core of tissue is removed with. a large
number of Ki - 67 positive cells, vascular invasion or a cutting needle. As in the case of nay needle biopsy, the main
breached capsule. A malignant secretes excess of problem is sampling error due to improper positioning of the
Noradrenaline than adrenaline whereas benign have higher needle, giving rise to a false negative test result."
adrenaline levels. .
113. (3)
108. (3) Macro calcification :
- Craniopharyngiomas are cystic tumors with area of Macroclacification on mammogram indicates benign lesion
calcification
- Originate in epithelial segments of rathke's pouch Benign Malignant
- Usually benign mass found in Sellar and suprasellar region Opacity Smooth margin III defined margin, Irregular
leading to Compression of pituitary optic tracts and third Low density stellate, speculated margin,
ventricle. Homogenous comet tail
- Radio graph shows area of sellar erosions with calcification Thin halo High density
In homogenous Wide halo
within or a above sella.
Calcification Macro calcification Microcalcification (<5mm in
- Most commonly seen in children but also present (+) in
(> 5 mmin diameter)
adulthood diameter)
- Also ass with suprasellar calcification and polyuria Breast Architectural distortion
- Causes growth retardation because of hypothalamic pituitary parenchyma
dysfunction. Nipple/areola Retracted Retracted
-Rx is subfrontal or transspenoidal excision with adjuvant RT if Skin Normal Thickened
total removal not possible. Coocer Normal Thickened, increased
liaaments number
109. (2) Ducts Normal Focal dilatation
2014 NBE ONELINER
Subcutaneous Normal Obliterated
retro mammary 118. (3)
space Simple Mastectomy
Since this patient is showing diffuse microcalcification on
Calcifications are the most common finding on mammogram. mammography, a simple mastectomy would be the most
Most of these are benign calcification. The calcification appropriate procedure.
patterns which are associated with high probability of Treatment of Ductal Ca in situ (DCIS)
malignancy are: DCIS carries a high risk for progression to an invasive cancer.
1. Clustered pleomorphic heterogenous macrocalcifications Schwartz writes - "The risk of invasive breast cancer is
There se small, usually less than 0.5 mm in diameter. vary in increased nearly five fold in women with DCIS that was
size, shape and density. originally detected, suggesting that DCIS is and anatomic
Often termed granular calcification. precursor of invasive ductal carcinoma."
2. Fine linear or fine branching calcifications. The treatment of intraductal lesion is controversial. It can be
These calcifications are often seen in comedocarcinoma . tit by either
Architectural distortion - Simple mastectomy
The structures of the breast are normally directed towards the - Breast conservation with wide excision (Lumpectomy) with
nipple. Disturbances in this normal orientation of tissue radiation therapy.
towards the nipple, especially the tethering or pulling in of Choice of operation depends on both surgeon and patient
structures to a point away from the nipple, is termed preference, however, following points must be kept in mind.
architectural distortion. Simple mastectomy is the gold standard procedure with no
Architectural distortion is highly suspicious for breast need of radiation and less recurrence rate (~2%), but there is
carcinoma, although benign conditions such as radial scar, loss of breast, a big psychological trauma to the patient.
surgical scar, fat necrosis can have this appearance. Breast conserving procedure is better cosmetically but
recurrence rate is more.
114. (4) The patient also has to be monitored regularly both clinically
According to latest AJCC classification the TNM stage would and by mammography for any recurrent disease.
be : Those in favour of conservative procedure, however say that
-T3N3,Mo local recurrences of occur, can be successfully managed with
- Hence stage III salvage mastectomy.
Simple mastectomy is recommended for the following DCIS
115. (2) DCIS with evidence of widespread disease (involving two or
LCIS (lobular Ca in situ) I occurs bilaterally in 50 to 70% of more quadrants) Mammographically identified multicentric
cases, while DCIS occurs bilaterally in 10 to 20% disease or diffuse suspicious calciflcation. Persistent positive
of cases. margins after re-excision.
"Invasive lobular Ca is frequently multifocal, multicentric Unacceptable cosmesis to obtain negative margins.
bilateral. A patient not motivated to preserve her breast
Size greater than 40 mm
116. (4) Comedo appearance on histology Negative estrogen receptor
Increased incidence with prolonged breast feeding status
Paget's disease of the breast is a superficial manifestation of High grade tumor
an underlying breast carcinoma. For patients with contraindications to radiation therapy
The underlying breast carcinoma is Intraductal carcinoma. a) Prior radiation to the breast region
As already described, prolonged breast feeding has some b) Presence of collagen vascular disease (SLE, Sclerodernia)
protective role in breast Ca. c) First or second trimester pregnancy
Paget's disease of nipple : Axillary lymph node dissection is not necessary for DCIS.
- It presents clinically as a chronic eczematous eruption of Adjuvant Tamoxifen therapy is given to all DCIS patients.
nipple, which may be subtle but may progress to an ulcerated Treatment of lobular carcinoma in situ (LCIS)
weeping lesion. - LCIS is not an anatomical precursor of invasive disease like
- It is usually associated with an DCIS on invasive ductal PCIS rater it is considered as a risk factor for invasive breast
carcinoma. The carcinoma mayor may not be palpable. carcinoma (Which canb be either invasive or lobular type).
- Thus paget's disease is a superficial manifestation of the - The risk for invasive cancer is equal for both breast, so there
underlying ductal carcinoma. no benefit to excise LCIS.
- Pathogomic histological feature is present of paget's cells in - Treatment of LCIS is
the epithelium. Paget's cells are large pale vacuolated cells. - Observation with or without tamoxifen
Treatment is described ahead. - The goal of treatment is to detect the invasive cancer at an
early stage which will develop in about 25 to 35% of these
117. (2) women.
Blockage of subdermal lymphatics: - For patients who are unwilling to accept the increased risk of
Peaud's orange is produced due to obstruction of superficial breast cancer may be offered bilateral simple mastectomy.
lymph vessels by cancers cells. Also remember
This causes edema of the skin giving rise to an appearance - The majority of invasive cancer which subsequently develops
like that of the skin of an orange (Peaud's orange appearance) in LCIS are of ductal nature (~65%) not lobular.
2014 NBE ONELINER
- One more difference between LCIS and DCIS. malignant elements is made, re-excision of the biopsy site to
ensure complete excision of a tumour with a 1 cm margin of
119. (1) normal appearing breast-tissue is indicated.
All the standard hospitals in the world recommend breast b) T/T of large phyllodes tumour Simple mastectomy.
conservative tit for stage I and stage II cancers, but breast Remember, cystosarcoma phyllodes do not metastasize by
conservativesurge. Surgery is an underutilized process. Still lymphatic route, so axillary dissection is not needed.
mastectomy remains the more common treatment for breast Cystosarcoma phyllodes metastasize exclusively by blood
cancer and mastectomy radical mastectomy is the most borne route
commonly performed procedure for Breast cancer. Other question on cystosarcoma phyllodes.
- In stage IU and II - Pathologically it is similar to fibroadenoma that is,
- Go for breast conservation surgery as the answer. proliferation of both duct and stroma is seen.
- Stage III - The tumor displays a range of behaviour from benign like
- Most of the stage III carcinomas can be managed by palliative fibroadenoma to frankly malignant
therapy. Curative therapy can be offered only in some cases. - Peak incidence is in the fifth decade, exceedingly rare in
- Stage IV adolescents.
- Palliation is all that is possible in stage IV carcinoma. - The excised specimen is firm and rubbery and may separate
Multicentricity means occurrence of a second breast cancer into leaf like structures therefore the term (phyllodes means
outside the breast quadrant of the primary cancer (cf : leaf Iike)
Multifocality refers to the occurrence of a second cancer within They usually present as a large growth which is nobile and
vicinity or in the same breast quadrant as the primary cancer). commonly there is a history of rapid growth.
T4b Breast tumor implies - tumor of any size with skin
involvement (cf : T4a implies tumors of any size with chest 121. (3)
involvement) Both T 4a and T 4b are relative contraindication 122. (2)
for Breast Conservative Surgery. (a) Cystisarcoma phyllodes : .
So we are left with two options T1 breast tumor and This tumor occurs predominantly in premenopausal women
Extensive in situ cancer around 40 years of age. However these may occur in
T1 breast tumor (means tumor s 2 cm in size) adolescents too. It presents as rubbery mass and on inspection
BCT is done in T1 tumors (unless some contraindications are it shows big swelling with tense glistening skin over it.
present) (b) Fibrocystic breast disease:
Extensive is situ cancer Fibrocystic disease is also rare in adolescent. It is usually seen
- Here the examiner is talking about extensive intraductal in women of 20-40 years of age. It can be easily ruled out as it
component - EIC presents with multiple masses associated with pain and
- EIC is pattern of DCIS that is seen associated with invasive tenderness.
carcinoma. (c)Early Carcinoma
- The above mentioned book writes - 'At our institution EIC Carcinoma is rare in adolescents. Carcinoma can be easily ruled
positive tumors are defined as invasive carcinomas that show out as it presents with hard lump.
the simultaneous presence of prominent DCIS within the (d) Virginal hypertrophy:
tumor (usually comprising 25% or more of the area of the Although it often presents as bilateral breast enlargement, yet
tumor) and DCIS beyond the edges of the invasive tumor. Also the typical history of rapidly developing breast enlargement
included among EIC positive cases are tumors that are and similar consistency of both the breast makes the diagnosis
predominantly DCIS with one or more foci or microinvasion'. of virginal hypertrophy likely. Virginal hypertrophy is a
The significance of EIC positive tumors is that massive sensitivity of the breast to oestrogenic hormones.
In these tumors the intraductal involvement is more extensive. Treatment: Reduction Mammoplasty
than can be appreciated clinically or at the time surgery. Frequency of Breast masses in adolescents
Therefore EIC positive patients who undergo a limited - Fibroadenma.- 54%
resection of the clinically evident tumor frequently have - Virginal hypertrophy - 13%
considerable residual DCIS in the vicinity of the tumor site. - Fibrocystic or proliferative breast lesson - 24%
This earlier led to increased recurrence in EIC positive tumors - Cystosarcoma phyllodes - 2%
managed by BCT.
But, now these tumors are not a contraindication for BCT, if 123. (4)
microscopically negative margins can be obtained. More Causes of Massive Enlargement of the Breast
careful mammographic and pathological evaluation is 1. Benign hypertrophy (usually bilateral)
necessary on EIC positive tumors to define the extent of the 2. Brodie's disease (Cystosarcoma phylloides)
lesion and to determine the adequacy of excision. 3. Giant fibroadenoma
But if one answer is to be chosen, it would be option (a) that is 4. Sarcoma
T1 breast tumor . 5. Colloid Carcinoma
6. Filarial elephantiasis
120. (1)
Treatment of cystosarcoma phyllodes 124. (4)
a) TIT of small phyllodes tumour It should be locally excised Approximately 2 percent of American women who develop
with an obvious 1 cm of normal breast tissue. After excision, carcinoma of the breast will be pregnant at the time of
when the diagnosis of a phyllodes tumor with suspicious
2014 NBE ONELINER
diagnosis. The therapeutic approach to these patients has Epidural hematomas are typically caused by a tear of the
changed considerably in recent years. Though changes in the middle meningeal artery or vein, or a dural venous sinus.
breast that occur during pregnancy often lead to a delay in Ninety percent of epidural hematomas are associated with
diagnosis of breast carcinoma, there is no convincing evidence linear skull fractures, usually in the temporal region. Only 2
that breast carcinoma in pregnant women behaves differently percent of patients admitted with craniocerebral trauma suffer
or its histologically different from that in non-pregnant epidural hematomas. The lesion appears as a hyperdense
women. Furthermore, when patients are matched for age and biconvex mass between the skull and brain on CT scan. Clinical
stage of disease, no significant differences in survival rates are presentation is highly variable and outcome largely depends
found. The majority of breast cancers in these patients as with on promptness of diagnosis and surgical evacuation. The
most premenopausal patients, is estrogen receptor negative typical history is one of head trauma followed by a momentary
and not hormonally sensitive. Therefore elective termination alteration in consciousness and then a lucid interval lasting for
of pregnancy is generally longer indicated to decrease estrogen up to a few hours. This is followed by a loss of consciousness,
stimulation of the tumor. Since radiation exposure endangers dilatation of the brainstem and death. Treatment consists of
the fetus and as there is no evidence that general anesthesia temporal craniectomy.evaluation of the hemorrhage, and
and nonabdominal surgery increases premature labor, control of the bleeding vessel. The mortality of epidural
modified radical mastectomy is recommended for stage I or II hematoma is approximately 50 percent.
carcinoma (tumor less than 4 cm in diameter). Patients in later
stages of pregnancy, however, can start radiation therapy 129. (4)
shortly after delivery, and some of them may be candidates for CSDT : "Posttraumatic epilepsy is more likely to develop if the
breast conserving surgery and adjuvant radiotheraphy. duramater was penetrated and generally manifests within 2
Chemotherapy does not appear to increase the risk of years following the injury."
congenital malformation when given in the second or third Harrison : The superficial cortical scars that evolve from
trimester of pregnancy. Patients who require adjuvant contusions are highly epileptogenic and may later manifests as
chemotherapy during the first trimester may opt for a seizures, even after many years."
therapeutic abortion, however, since there is a slightly
increased risk of fetal malformation in that circumstance. 130. (1)
"Extradural haemorrhage is an accumulation of blood between
125. (2) the skull and the dura. In 90% of cases, bleeding is from a
Paget's disease of the breast is unrelated to Paget's bone branch of the middle meningeal artery".
disease. It represents a small percentage (1 to 3 percent) of all Also remember
breast cancers and is through to originate in the retroareolar Subdural haemorrhage, commonest source is
lactiferous ducts. It progresses toward the nippleareola communicating veins b/w the cerebral cortex and superior
complex in most patients, Where it causes the typical finding of saggital sinus.
nipple eczema and erosion. Up to 20 percent of patients with Intracerebral haemorrhage, commonest source is
Paget's disease have an associated breast mass, and these lenticulostriate arteries in the region around the basal ganglia*
patients are more likely to have involvement of axillary nodes. and intemal capsule. It is frequently d/t rupture of Charcot-
Nipple areolar disease alone usually represents in situ cancer; Bouchard aneurysms in the lenticulostriate arteries.
these patients have a 10 year survival rate of over 80 percent. Subarachnoid heamorrhage - MC cause is rupture of Berry
In contrast, if Paget's disease presents with amass, the mass is aneurysms.
likely to be an infiltrating ductal carcinoma. The generally
recommended surgical procedure for Paget's disease is 131.(2)
currently a modified radical mastectomy. The validity of breast Chronic subdural haematomas. These haematomas are
saving surgery and adjuvant radiation therapy for patients most common in infants and in adults over 60 years of
without an associated mass is under investigations. age. They present with progressive neurological deficits
more than 3 weeks after the trauma. Often, the initial
126. (1)
The most common are sodium disturbances, which head injury has been completely forgotten and the
carfocurboth spontaneously and as a consequence of diuretic pathology has been attributed to either dementia or a
therapy to control raised intracranial pressure. Both hypo and brain tumour until patients are scanned. The initial
hypernatremia can further compromise consciousness and haemorrhage may be relatively small or may occur in
injudicious rapid correction can induce pontine myelinolysis. elderly patients with large ventricles or a dilated
subarachnoid space. Membranes derived from the dura
127. (2) and arachnoid mater encapsulate the haematoma, which
Burst temporal lobe is the term sometimes used. to describe remains clotted for 2-3 weeks then liquefies. The acute
the appearance of confusional intracerebral haematomas, clotted blood initially appears white on a CT scan. As it
bleeding out into the subdural space from a disrupted cortical
liquefies, it slowly becomes black. Thus, there is a point
surface. The blood follows the subdural space over the
convexity of the brain and appears as a concave hyperdense in time where it appears iso-dense with brain and all that
collection. can be seen is apparent inexplicable shift on an otherwise
normal CT. These collections can then either resolve or
128. (3) increase in size from osmotic effects or repeated small
2014 NBE ONELINER
bleeds. They are evacuated by drilling burr holes over the 138. (3)
collection and washing it out with warmed saline. Lesch nodules (pigmented hamartomas of the iris) and
optic gliomas are two of the criteria that can be used to
132. (3) diagnose neurofibromatosis type I. Other characteristics
Steroids help to reduce peritumoral oedema in order to include plexiform neurofibromas and autosomal
reduce mass effect. Surgery is appropriate if there is a dominant inheritance.
solitary surgically accessible lesion and no systemic
spread, particularly if the primary site is unknown and 139. (1)
the histological diagnosis in doubt. Radiotherapy is used These findings are suggestive of hypercortisolism. This
to treat multiple metastases and following resection. may be secondary to a number of sources including
Stereotactic radiosurgery has been used in the treatment adrenal tumors, exogenous steroid use, pituitary tumors,
of metastases smaller than 2 cm. with results similar to and malignancies at other sites (e.g. small cell carcinoma
those of surgery. of the lung). Among the choices, Cushing's disease
defined as hypercortisolism secondary to an
133. (2) adrenocorticotropic hormone (ACTH)-producing
The treatment of choice for medulloblastoma is surgical pituitary adenoma is the most likely diagnosis.
debulking and radiation therapy, which results in a 5
year survival of 56%. Chemotherapy with carmustine is 140. (4)
sometimes used in relapse, but is ineffective as an initial A suprasellar calcification is typical of
treatment modality. Medulloblastomas are the most Craniopharyngioma
common brain tumors in children. They are associated Polyuria is d/t diabetes insipidus caused d/t
with the roof of the fourth ventricle, not the floor. compression by the tumor over pituitary.
Subarachnoid spread is seen in 35% of cases at time of - "Clinically, patients with suprasellar tumors typically
diagnosis. present with diabetes insipidus, endocrinopathy" .

134. (3) 141. (4)


Meningiomas are slow growing tumors that often present Dandy walker malformation is a cystic expansion.of 4th
with comR!;lintlif9t chronic headache. The olfactory ventricle in the post fossa due to failure of roof of 4th
groove is a common location for meningiomas ventricle development.
- 90% patients have hydrocephalus.
135. (2) - As anomalies include agenesis of post. cerebellar vermis
Patient with Oligodendrogliomas classically present with and corpus callosum.
a long history of seizures. These tumors are often - Most children have evidence of long tract signs/
calcified on computed tomography (CT) scan and most cerebellar ataxia & delayed motor and cognitive mile
commonly occur in the frontal lobe. stone.
- Management is shunting the cystic cavity in case of
hydrocephalus.
136. (3)
The finding of bilateral vestibular schwannomas 142. (2)
involving cranial nerve VIII is considered diagnostic for Berry aneurysm is me type of intracranial aneurysm
neurofibromatosis type II. - Congenital weakness of wall
- Increased risk with Ehler Dan los /NF1, marfan's /
137. (2) APKD /fibromuscular dysplasia.
Mannitol an osmotic dehydrating agent, Works by - Predisposing factor - HTN and smoking.
drawing water from parts of the brain with an intact - Rupture usually occurs at apex of sac leading to SAH or
blood-brain barrier. If this is disrupted, as in a cerebral Intraparenchymal hrg or both
contusion, mannitol can leak out into the brain and - 90% occurs in Ant circulation.
potentiate the mass effect. In head injuries it should - The wall is made up of thickened hyaline intima.
therefore, only be administered after consultation with a Rx - Embolisation /clipping of the Sac.
neurosurgeon. It becomes ineffective when brain
osmolality becomes iso-osmolar with that of the serum. 143. (3)
Steroids work by stabilizing the blood-brain barrier and Fronal lobe lesion tends to present with personality
reducing oxygen radical injury. Barbiturates give rise to change, gait ataxia and urinary incontinence, Pathological
vasoconstriction. Frusemide reduces ICP by joking (Witzel Schultz syndrome). Parietal lobe lesions
reducing cerebral oedema and CSF production. are associated with sensory inattention, dressing apraxia,
asterognosis [a graphia, Lt-to-Rt Disorientation and
2014 NBE ONELINER
finger agnosia (Gertzman's Syndrome) if and dominant Cryoprecipitate
side affected)]. Temporal lobe lesions cause hemiparesis, < - 30C 1 yr
- Frozen
contralateral superior quadratatopia and memory 20 - 24C 4 hrs
- Thawed
disturbances; dysphagia. Occipital lesion associated with Stored blood lacks functional platelets after 24 hrs.
incomplete contralateral Homonymous hemianopia.
148. (3)
144. (3) Symptoms of hemolytic transfusion in
Secondary haemorrhage occurs after 7-14 days & is due Conscious patient Unconscious patient
to infection & sloughing of part of the wall of an artery. - Sensation of heat and oain in the - Diffuse Bleedina and
(1), (2) & (4) are seen with reactionary limbs
- Headache severe. Hypotension
- Tachycardia
- Precardial and lumbar pain - Urticarial rash
145. (3) - Tachv cardia and suffused face - Raising airway pressure
When oliguria occurs postoperatively, it is important to - Oliguria and Hemoalobinuria on IPPV
differentiate between low output caused by the
physiologic response to intravascular hypovolemia and 149. (3)
the caused by acute tubular necrosis. The fractional Shock is defined as in adequate perfusion to maintain
excretion of sodium (FE) is an especially useful test to aid normal organ function. Thus, the goal in the Rx of shock
in this differentiation. Values of FE <1 percent in an is restoration is of adequate organ perfusion and tissue
oliguric setting indicate aggressive sodium reclamation Oxygenation.
in the tubules; values above this suggest tubular injury. Urine output is quantitative and relatively reliable
The fractional excretion is a simple calculation: (urine Na indicator of organ perfusion Adequacy of resucitation can
X serum creatinine) + (serum sodium X urinary be determined by urine output and blood pH.
creatinine). In the setting of postoperative hypovolemia,
all findings would reflect the kidney efforts to retain 150. (2)
volume: the urine sodium would be below 20 meq/L, the Platelet concentrate is required for patients with
urine chloride would not be helpful except in the thrombocytopenia.
metabolically alkalotic patient, the serum osmolality FFP can be given in any of the congenital clotting factor
would be over 500 mOsm/kg, and the urine /serum deficiency diseases, especially Christmas disease (factor
creatinine ratio would be above 40. IX deficiency) or haemophilia B. Cryoprecipitate is a very
rich source of factor VIII.
146. (4)
The risk of V. fibrillation increases at temp below 28C 151. (4)
and Cerebral metabolism is decreased by 6-7% of per The following are causes of retroperitoneal fibrosis also
1drop in temp. Cardiac arrest occurs at 20C. The known as retroperitoneal fasciitis or chronic
Patients should be rewarmed for 24 hrs at room temp of retroperitoneal fibroplasias.
25-33C severe Hypothermia (< 28C) should be treated 1) Hodgkin's disease
as life threatening emergency. At temp < 30C, heart is 2) Ca breast
unresponsive to defibrillation and ionotropes so 3) Ca colon
medicines are best withheld until rewarming have been 4) Methysergide used for migraine headaches
achieved. 5) Membranous glomerular disease
6) Inflammatory bowel disease
147. (4) 7) Leaking aortic aneurysm
Half life of platelets is 9 days
RBC's adult = 120 days 152. (4)
V and VIII = 6-12 hrs. There is a narrowing where the ureter is coursing in the
Blood component Temp for Shelf life bladder wall & not before entering the bladder wall.
Whole blood 1 - 6C 35 days
Packed cells - 42 days 153. (3)
Platelets 20-24 C 5 days Stretching of renal capsule gives pain in the loin which
Leucocyte conc. 20 - 24C 24 hrs deep seated sickening ache.
FFP < - 30C 1 yr Ureteric colic gives pain which radiates from loin groin,
the more distal the stone, the more the pain radiates to
Thawed plasma 1 - 6 C 24 hrs
the groin.
Albumin Room Temp 3 yrs Severe inflammation of bladder causes wrenching
discomfort at end of micturation. Urethral pain is
2014 NBE ONELINER
scalding or burning during voiding felt in the vulva or Any renal calculi that is 4 mm or less in size, is allowed
penis. some time (approx. 4 weeks) to pass spontaneously.
Indications for surgical removal of a ureteric calculus -
154. (1) Stone is too large to pass spontaneousiy
A stone in the ureter nearly aIways has its birth in the - Urine is infected
kidney. - Repeated attacks of pain and the stone is not moving
When the stone is in the intramural ureter the pain is - Stone is enlarging
referred to the tip of the penis. - Stone is causing complete obstruction
Urine is not always infected. If infection is present, then it - Stone is obstructing solitary kidney or there is bilateral
is an indication for surgical removal of the calculus. obstructions.
Otherwise, expectant treatment is appropriate for small
stones that are likely to pass naturally. 158. (2)
Risk factors for squamous cell ca of bladder
155. (4) Schistosoma haematobium
It is a well known fact that cyanide is produced from - it's a risk factor for both TCC & sq. cell ca, but more for
heated uric acid. sq. cell ca.
In Holmium YAG laser lithotripsy occurs, primarily Chronic irritation from urinary calculi, long term
through a photo thermal mechanism that causes stone indwelling catheters, Chronic urinary infections
Vaporization. Bladder diverticula
So production of cyanide when Holmium: YAG laser is
used for treatment of uric acid stones is considered a 159. (1)
potential side effect of this therapy. This has been Vesical caluli are common in some parts of the country,
reported in vitro e.g. Rajasthan, Andhra Pradesh and some north-eastern
This event has not been observed to produce any states. These stones are usually single and composed of
adverse event in treated patients. This is probably due to ammonium and urate and calcium oxalate (ie
the fact that ablation site is continuously flushed with radioopaque stones). These calculi usually occur in boys
liquid during the procedure. below 5 yrs. of age.
More on Holmium YAG laser. Most bladder stones contain calcium and are radiopaque,
- Wavelength used is 2100 nm. cystoscopy is confirmatory because stones in the pelvis
- Currently the HO : YAG laser is the most effective and seen on plain film may prove to be extravesical.
versatile Intracorporeal Lithotriptor with good margin of Treatment is usually transurethral Cystolitholapaxy."
safety. Primary bladder stone is one that develops in sterile
- HO-YAG Laser Lithotripsy occurs primarily through a urine. If often originates in a kidney and passes down the
photothermal mechanism that causes stone vaporization. ureter to the bladder, where it enlarges.
- They have the ability to fragment all types of stones Secondary bladder stones occur in presence of some
regardless of the composition of the stones. infection, bladder outflow obstruction, impaired bladder
- An important side effect of the laser is that it will melt emptying or a foreign body such as nonabsorbable
the wires of a basket or guide wire if it is fired directly sutures, metal staples or catheter fragments. Secondary
making extraction of the wire or the basket very difficult. bladder stones are more common than primary.
Treatment of Vesical calculus :
156. (4) Vesical calculus is removed usually by per-urtheral
For staging the disease as, T1 or carcinoma in situ, or litholapaxy.
even T2, histological examination is required as to the Contraindication to per-urtheral litholapaxy.
invasion of lamina propria or muscle. Biopsy specimen A very large stone, a contracted bladder, a urethral
can only be obtained by cystoscopy; hence it is the right stricture that cannot be dilated sufficiently When the
answer. patient is below 10 years of age.
Alternative procedures: Suprapubic lithotomy,
157. (4) Percutaneous suprapubic litholapaxy.
"Ureteroscopic stone extraction is highly efficacious for
lower ureteral calculi" - Smith's Urology. 160. (1)
A ureteroscope is a long endoscope that can be passed Balkan nephropathy Transitional cell tumours of the
transurethrally across the bladder into the ureter. The upper urinary tract have a very high incidence in certain
stones can be caught in baskets or endoscopic forceps. areas of the Balkans. The same population has a high
Those that cannot be removed via baskets or endoscopic incidence of a form of primary nephropathy. The
forceps, are fragmented using ureteroscopic lithotrities. causative agent has not been identified with certainty but
there seems to be an association with the consumption of
2014 NBE ONELINER
grain products stored in a damp environment. Tumours 167. (2)
that develop against a background of Balkan The patient with incomplete bladder emptying & good
nephropathy should be treated by nephron-sparing capacity may be managed by means of clean intermittent
surgery in view of the impaired overall renal function. catheterisation (CISC).
The patient with complete bladder emptying &
161. (3) reasonable capacity with normal upper tracts may be
In case of extra peritoneal rupture of bladder a managed by means of condom drainage.
cystogram shows extravasation of contrast material into Patients with poor emptying, low capacity & upper tract
the pelvis around the base of bladder. The characteristic dilatation require addditional treatment.
'TEAR DROP' deformity represents bladder compression
by a large pelvic haematoma. 168. (1)
The main complication is uretero ileal stricture which
162. (2) can be limited by spatulation of the distal ureters & an
"Schistosomiasis - is the commonest cause of calcification end-to- end anastomosis.
in the wall of bladder. Thin curvilinear calcification Stenosis at the ileocutaneous site is less frequent & a
outlines a bladder of normal size, and shape. Calcification short isoperistaltic conduit limits the formation of a
spreads proximally to involve the distal ureters residual urine volume to reduce infection & avoid the
(appearing as two parallel lines) in 15%" problems of reabsorption of urine.
Schistosomiasis causes calcification in the wall of the
bladder which outlines the bladder. A full bladder will 169. (1)
appear as a fetal head in pelvis. The patient is suffering from genuine stress incontinence.
It is usually found in multiparous women with a history
163. (2) of difficult labour often accompanied by the use of
Urodynamic study is done to distinguish genuine stress forceps. It can be found in normal young women who
incontinence (due to sphincter weakness) from detrusor indulge in competitive trampooning & also in patients
instability in women and investigation of incontinence in with epispadias. The symptoms may change with the
general. menstrual cycle.

164. (2) 170. (2)


In the first year of life, the bladder is closed following Cis-platinum plus gemcitabine given before
osteotomy of both iliac bones just lateral to the sacroiliac (Neoadjuvant) radical cystectomy has been shown to be
joints. Later, reconstruction of the bladder neck and of benefit.
sphincters is required.
171. (4)
165. (1) Full distension of the bladder is needed if searching for a
There is intermittent, painless, terminal haematuria seen. diverticulum. With inadequate distension of the bladder,
Sandy patches on cystoscopy are the result of calcified the mouth of the diverticulum is closed with epithelium
dead ova with degeneration of the overlying epithelium. thrown into radiating pleats.
Considerable calcification of this nature is seen on Ivu may give information regarding size of diverticulum.
radiograph. Fibrosis is mainly the result of secondary (3) is done only during video urodynamic investigation &
infection. The capacity of the bladder becomes much will also give information about the emptying
reduced and contracture of the bladder neck may be characteristics of the bladder & diverticulum
found. Carcinoma is a common end result in grossly
infected bilharziasis is of the bladder which has been 172. (3)
neglected for years. Urinary calculi, especially vesical and The most common sites for superficial tumours are the
ureteric, occur more frequently when bilharziallesions of trigone & lateral walls of the bladder.
the bladder are present.
173. (4)
166. (2) Oestrogen deficiency, which may give rise to lowered
Ca bladder is common due to constant irritation of local resistance seen in menopausal females predisposes
bladder wall. to UTI. Other causes are:
In males, the completely epispadiac penis is broader and 1. Meatal stenosis
shorter than normal and bilateral inguinal herniae may 2. Bladder diverticulum
be present. There is uretero vesical efflux and 3. Presence of calculus, foreign body or neoplasm.
incontinence but no ventral curvature of penis. 4. Diabetes
5. Immunosuppression
2014 NBE ONELINER
Chronic lesions of Granuloma inguinale may become
174. (3) grayish, especially the edges, where, after months or
Carcinoma is a common end result in a grossly infected years, malignant change may develop. The ulcerated area
bilharziasis of the bladder which has been neglected for bleeds' on touch & without treatment healing is only
years. It usually commences, not in a papilloma, but in an partial & keloid is common.
ulcer & is therefore a squamous cell carcinoma (due to Genital warts are sexually transmitted & may complicate
metaplasia). HIV lnfection.

175. (4) 181. (1)


Frequency-dysuria syndrome or urethral syndrome is By the age of 16 years, irreversible destructive changes
more common in women & consists of (1), (2) & (3). have occurred in the testis which halt spermatogenesis &
Carcinoma-in-situ, tuberculosis & interstitial cystitis limit the production of androgens to around half of the
should be excluded. normal output.
General hygienic measures are applied as part of
treatment. 182. (3)
This is a verrucous carcinoma affecting the penis. It is
176. (4) locally destructive & invasive but appears not to spread
The physiological adhesions between the foreskin & to lymph nodes to metastasise. Treatment is surgical
glans penis may persist until 6 years or more of age. excision.
True phimosis consists of scarring of the prepuce, which
will not retract without fissuring. (1) Occurs later in life 183. (4)
in which the normally pliant foreskin becomes thickened Priapism is idiopathic in 60% of cases, while the
& will not retract. remaining 40% of cases are associated with diseases (eg
leukemia, sickle cell disease, pelvic tumors, pelvic
177. (4) infections, penile trauma, spinal cord trauma, or use of
Spontaneous remission is seen in 50% cases of the medications. Currently, intracavernous injection therapy
disease. If remission does not occur, p-amino benzoic for impotence may be the most common cause
acid powder or tablets or vitamin E tablets may be tried
for several months. 184. (3)
A number of operative procedures have been used after Gonorrhoea is associated with infection of peri-urethral
excision of the plaque. glands and discharge per urethra.
Placement of non absorbable sutures opposite the plaque Skin conditions like lichen planus & psoriasis affect the
without its excision is Nesbitt's operation. penis. Drug hypersensitivity reactions can affect the skin
of the penis. Balanoposthitis is associated with penile
178. (2) cancer, diabetes & phimosis. Monilia infections are also
Giant condylomata acuminata is a precancerous quite common under the prepuce.
dermatologic lesion which is cauliflower like arising from
the prepuce or glans. It is difficult to distinguish from 185. (1 )
well - differentiated squamous cell Ca. Four chemotherapeutic agents demonstrate activity
Bowen disease is a squamous cell Ca-in-situ involving the against penile carcinoma. Bleomycin, methotrexate,
penile shaft. It appears as a red plaque with cisplatin & 5- FU.
encrustations. Erythroplasia of Queyrat or Paget's
disease of the penis, is a velvety, red lesion with 186. (2)
ulcerations that usually involve Non filarial elephantiasis can result from fibrosis of the
the glans. lymphatics due to Lymphogranuloma venereum.

179. (4) 187. (2)


Patients who initially have clinically negative nodes but This is seen in middle aged and old men. Patients present
in whom clinically palpable nodes later develop should with complaints of painful erection, curvature of the
undergo a unilateral ilioinguinal node dissection. penis, and poor erection distal to the involved area.
Patients who have inoperable disease & bulky inguinal The patient has no pain when the penis is in the non-
metastases are treated with chemotherapy. In some erect state.
cases regional radiotherapy can provide significant The condition has been noted in association with
palliation. Dupuytren's contracture of the tendons of the hand, in
which the fibrosis resembles that of Peyronie's disease
180. (2) when examined microscopically.
2014 NBE ONELINER
Spontaneous remission occurs in about 50% of cases. Seminoma is the most common germ cell tumor in
bilateral testicular tumors, while malignant lymphoma is
188. (2) the most common bilateral tumor of the testis
Patients with bulky retroperitoneal disease (> 3 cm
nodes or 3 or more 1- cm cuts on CT scan) or metastatic 195. (3)
NSGCT are treated with primary platinum-based Partners should be warned that they should continue
combination chemotherapy following orchiectomy. If with their contraceptive precautions until the success of
tumor markers normalize and a residual mass is the operation has been confirmed by semen analysis
apparent on imaging studies, resection of that mass is performed 12-16 weeks after surgery.
mandatory, because 20% of the time it will harbour
residual cancer, 40% of the time it will be teratoma and 196. (2)
40% of the time it will be fibrosis. (1),(3) & (4) are false statements.
The seminal vesicle, which is affected, & the reservoir of
189. (1 ) infection feel indurated & swollen on PR examination.
Torsion of testis, which is the condition in question, can Lax hydrocele occurs in 30% of cases. Cold abscess is
take place spontaneously. Testicular torsion is most formed in neglected cases over period of time.
common between 18 and 25 years of age and a few cases
occur in infancy. Elevation of the testis reduces the pain 197. (2)
of epididymo-orchitis and it makes it worse in torsion. The presence of contralateral atrophy or
Arrangements should be made for early operative ultrasonographic microlithiasis in patients with
fixation to avoid recurrent torsion if gentle manipulation testicular tumors warrants contralateral biopsy. If
fails. diagnosed CIS is usually treated by external beam
radiation therapy.
190. (1 )
By doing a transscrotal biopsy there will be metastatic 198. (4)
spread of testicular tumor along the needle track. Hence, For patients with bulky seminoma & any seminoma
contraindicated. Scrotal U/S will assess primary associated with elevated AFP should receive primary
testicular tumor rapidly. chemotherapy. Some of the successful regimens include
Inguinal exploration with cross clamping of the cisplatin, etoposide & bleomycin (PEB), vinblastine,
spermatic card vasculature and delivery of the testis into cyclophosphamide, Dactinomycin, bleomycin, & cisplatin
the field is the mainstay of exploration for a possible (VAB-6); & cisplatin & etoposide.
testis tumor. If cancer cannot be excluded by
examination of the testis, radical orchiectomy is 199. (2)
warranted. Metastasis to the testis is rare. The most common
primary site is the prostate, followed by the lung,
191. (3) gastrointestinal tract, melanoma & kidney
Circumcision soon after birth confers almost complete
immunity against carcinoma of the penis. Later 200. (2)
circumcision does not seem to have the same effect and The characteristics of a normal semen sample are:
Moslems circumcised between the ages of 4 and 9 years Ejaculate volume - 1.5 - 5.5 ml
are still liable to the disease. (1), (2) & (4) are true Sperm concentration >20 x 106 sperm I ml.
Motility> 50%
192. (1) Forward progression: 2 (scale 1-4)
Leydig cell tumors are the most common non-germ cell Morphology:> 30% WHO normal forms (> 4% kruger
turners of the-testis & account for 1-3% of all testicular normal forms)
tumors. Sertoli cell tumor comprises less than 1 % of all
testicular tumors. Gonadoblastoma comprise 0.5% of all 201. (3)
testicular tumors The best treatment for gastric cancer is surgery. It's a
cancer so it has to be radical surgery. Since the tumour is
193. (2) involving distal stomach and is considerably smaller we
AFP is not elevated at all in seminoma submit of HCG is have the liberty to spare some stomach. So we go for
elevated in upto 7% of seminimas subtotal radical gastrectomy. It would be accompanied
by D2 - Resection because node along named vessel
194. (4) (celiac) is involved and needs to be removed.

202. (4)
2014 NBE ONELINER
The description in question is a classic presentation of emptying of hyperosmolar contents into the small bowel
Acute Gastric Distension. It is commonly seen after which then evokes a sympathetic response. It will be
splenectomy and surgeries in region of fundus of aggravated by more food. Patient has got raised
stomach. The management is usually conservative and hematocrit due to fluid shift.
includes option 1, 2, & 3 surgery is not required. Due to
vomiting patient may have electrolyte disturbance which 210. (4)
need to be corrected accordingly. Best treatment for bleeding is to stop the bleeding which
in case of duodenal ulcer is by ligation of the bleeding
203. (4) vessel, which is most commonly the gastroduodenal
Commoner variety of gastric volvulus is organoaxial. It is artery. Truncal vagotomy and andpyloroplasty is
usually associated with hiatus hernia, not Bochdalek treatment for recurrent ulcers.
hernia. Investigation of choice is Barium swallow.
Endoscopy is infact dangerous, it may lead to perforation. 211. (1)
Transverse colon commonly herniates into the thoracic Commonest cause of duodenal blow out leading to
cavity along with the stomach. duodenal fistula is post gastrectomy (Bilroth II). Other
causes are post operative after surgeries on biliary tract,
204. (4) duodenum; pancreas, right colon, aorta and kidney. Only
In this case we have involvement of liver and pancreas 15% are as a result of trauma, perforated ulcers and
along with Para aortic lymph node involvement all of cancers.
which indicate inoperability. The best palliation for
carcinoma stomach is again surgery. Radiotherapy has 212. (2)
poor role in carcinoma stomach. Palliative CT would be Upper GI bleed is due to either bleedipg peptic ulcer or
the option if patient is not fit to undergo any surgery. oesophageal varices. In an acute episode due to
esophageal varies endoscopic sclerotherapy ligation is
205. (3) effective. If this fails then one tries balloon tamponade.
Most common benign tumour of stomach is epithelial Vasopressin causes splanchnic vasoconstriction and
polyp. Adenomatous polyp is a premalignant condition reduces bleeding in an acute episode. Blockers are used
and is just one of the five types of gastric polyps for preventing the recurrence of bleeding. They are not
(Hyperplastic, Adenomatous. Hamartomatous, used in acute setting.
Inflammatory and Heterotopic)
213. (3)
206. (4) Diarrhea occurs in postgastrectomy patients. Calcium
Treatment of primary gastric lymphoma is controversial. deficiency, vitamin B12 deficiency and steatorrhoea occur
Bailey says surgery should be the most appropriate due to malabsorption.
option and chemo reserved only for those with systemic
disease. The indication for surgery is bleeding, 214. (1)
perforation, obstruction, fistula formation and failure of Malignant change does not occur in duodenal ulcer. In
chemotherapy. case of perforation patient will present with acute
abdomen. The symptomatology in question is typical of
207. (3) gastric outlet obstruction which is most commonly
There has been a proximal migration in the site of associated with long standing peptic ulcer disease and
stomach cancer, but still all over the world about 40% gastric cancer. Endoscopic dilatation may be tried, but
tumors occur in antrum, 30% in fundus and 25% at the operative surgery is usually required.
cardia. Tumours are more common on lesser curvature
than the greater curvature; Also Ca stomach is commoner 215. (3)
in lower socioeconomic group and in black males. The vomiting of hydrochloric acid results in
hypochloremic alkalosis. Initially urine has a low chloride
208. (4) and high bicarbonate content reflecting primary
Hypertrophic pyloric stenosis causes metabolic alkalosis metabolic abnormality. This bicarbonate is excreted
with paradoxical aciduria. It is commoner in first born along with sodium and so with time the patient becomes
male child. Ramsted's pyloromyotomy is the surgical progressively hyponatremic and more profoundly
procedure. Vomiting is non-bilious. dehydrated. Because of dehydration a phase of sodium
retention follows and potassium and hydrogen ions are
209. (1) excreted in preference. This results in urine being
Early postcibal (dumping) syndrome occurs within an paradoxically acidic and hypokalemia ensues.
hour (usually 30min) of meals. It is due to rapid gastric
2014 NBE ONELINER
216. (2) Diagnosis is by Barium meal and hypotonic
The patient has presented with a recurrent disease. So duodenography.
the main aim of surgery now should be to have least Treatment initially conservative and is usually successful.
recurrence. Amongst the given options, option (2) has Operative treatment is Duodenojejunostomy
the least recurrence rate (only 1%), highly selective
vagotomy has least mortality and morbidity but maximal 221. (1)
recurrence rate (2-10%). Truncal vagotomy is not done Laparotomy and tube duodenostomy makes it a
in isolation, it is always accompanied by a drainage controlled fisula and feeding jejunostomy allows enteral
procedure. feeding. So option (1) is the best. Reanastomosis in this
scenario would lead to leak hence should not be
217. (2) attempted. Enteral
Duodenoduodenostomy is the best option. Duodenal feeding should always be preferred over parenteral
atresia occurs at the point of fusion between foregut and feeding.
midgut and therefore lies in neighbourhood of ampul!a of
Vater. It occurs due to failure of complete recanalisation 222. (4)
of duodenum in utero. Treatment is The band extends from 2nd part of duodenum of the right
duodenoduodenostomy which is constructed using most Paracolic gutter. This anatomical location of band would
dependant portion of proximal atretic segment to most commonly obstruct duodenum.
minimize stasis.
223. (2)
218. (1) Coil spring or stacked coin is seen on Barium Swallow in
Jejunoileal bypass was the first surgical approach to case of duodenal hematoma. Although characteristic of
morbid obesity and was effective in reducing weight. intramural duodenal hematoma. This finding is present
However the procedure is widely practiced because of only in 25% of patients with duodenal hematoma.
multitude of complications. Most serious of which is liver Operative exploration and evacuation othematoma may
disease. This was secondary to protein calorie be considered after 2 weeks of conservative therapy.
malnutrition and bacterial overgrowth in bypassed Most of duodenal hematomas resolve by conservative
segment. None of the patients operated for morbid management consisting of nasogastric aspiration and
obesity can maintain weight reduction unless he follows total parenteral nutrition.
a strict dietary regimen. Ulceration after roux-en-y
gastric bypass does not occur in antrum, it occurs in the 224. (1)
small intestinal part anastomosed to stomach. Duodenal atresia can be classified into
Type I (most common) - mucosal web with normal
219.(2) muscular wall (windsock deformity) Type II short fibrous
Secondary diverticuli are the ones which occur in 1st cord connecting two atretic ends
part of duodenum and they are usually the results of Type III (least common) - complete separation of atretic
scarring following duodenal ulcer. They are false ends
diverticula. 90% of primarily diverticula are solitary and An intrinsic obstructive web is known as windsock
80% are in second part of duodenum commonly on its deformity
concavity, in region of Ampulla of Vater. Most of them are
asymptomatic and found incidentally. 225. (2)
The most dangerous type of esophageal perforation
220. (4) amongst given options is Boerhaave's syndrome, because
Mesenteric arterial syndrome also known as superior here the esophagus perforates under pressure leading to
mesenteric artery syndrome results from compression of massive mediastinal spillage of contents. Iatrogenic
third part of duodenum between superior mesenteric perforation is the commonest type of esophageal
artery and aorta. This condition occurs most commonly perforation.
in young asthenic individuals, with females more
commonly involved than males. 226. (3)
Predisposing factors include Angiodysplasia is seen above 60 years of age. Ascending
Weight loss colon is the commonest site.
Supine immobilization Selective angiography is the best of investigaion and
scoliosis cannstigation and can also be used to embolize the
placement of body cast (cast syndrome) bleeder.
Symptoms include nausea, vomiting, abdominal Angiodysplasia is associated with arotic stenoses
distension, weight loss, postPrandial epigastric pain.
2014 NBE ONELINER
227. (4)
Abdominal actinomycosis usually presents as a right iliac 236. (4)
fossa mass 3 - 4 weeks after an appendicectomy. Lower gastrointestinal bleeding is defined as a bleeding
Penicillin is the drug of choice from a site distal to the ligament of Treitz.
"Hemorrhoids and anal fissure are the most common
228. (4) cause of lower G. I. bleeding however the bleeding is
Paralytic ileus is the commonest complication in typhoid, rarely massive",
which presents as a non-obstructive intestinal Angiodysplasia and diverticulitis are the other two
obstruction. Haemorrage is the leading symptom. common causes of bleeding and they usually present
A perforation usually occurs in the third week. with massive bleeding:
Here are the causes of acute lower GI bleeding :-
229. (4) Common
The fascial condensation which separates the rectum - Angiodysplasia
from the prostate is the Denonvillier's fascia. The - Diverticulosis
Waldeyer's fascia separates the rectum from the sacrum - Anorectal disease
and coccyx. Less common
- Neoplasia Carcinoma, polyps
230. (1) - Colitis Radiation, ischaemic, ulcerative
5 Fluorouracil in combination with leucovorin in the - Infective Enteric fever, amoebic ulcer, T.B.
chemotherapy of choice in rectal cancer. Newer drugs HIV related (gonorrhoea, CMV.)
like irinotecan and oxaliplatin have shown good promise. Rare
- Meckel's diverticulum
- Intussusception
231. (2)
It is important to confirm the absence of a mechanical 237. (2)
cause of the obstruction by colonoscopy or single Rectal polyps are most commonly seen in juvenile
contrast water soluble bariums enema. Colonoscopy is patient. The most common presentation of rectal polyp is
also the treatment of choice for decompression. It may painless bleeding per rectum. Meckel's diverticulum is
recur in 25% of cases. If colonoscopy fails, a cecostomy most commonly present as diverticulitis and this causes
may be required. pain in abdomen with bleeding P/r

232. (1) 238. (4)


Synchronous malignancy means two separate sites of the Not only a solitary but even multiple metastasis is not a
colon having a malignancy or the same tumour within 6 contraindication for surgery because surgery is also the
months. This is seen in 5% of cases. Hence, it is important best palliative measure.
to have a coloscopy for all patients of rectal or sigmoid If only a solitary liver metastasis (even upto 3 static
cancer in order to look for a synchronous cancer in the lesions) is present it can be later resected with good
caecum/transverse or ascending colon. prognosis. Carcinoma of the left side of the colon usually
of stenosing variety. The main symptoms are those of
233. (2) increasing intestinal obstruction, altered bowel habits;
It is usually a clockwise twist of the caecum - obstruction tenesmus and bleeding.
may be partial. Caecopexy or Caecostomy is the Anemia (severe and unyielding) is seen in right sided
treatment of choice. lesions.
Mucinous and 'signet ring' cell varities have poor
234. (1) prognosis.
Sigmoidoscopy done is a patient with acute diverticulitis Colorectal carcinomas are resistant to most
of colon shows inflammation of the mucosa. chemotherapeutic agents. Recently 5 FU and levamisole
The X-Ray abdomen in acute diverticulitis shows saw have been found to be of some use. They are used as
tooth appearance of the mucosa rigid sigmoidoscope adjuvant chemotherapy in Dukes stage C lesions.
could be passed upto 16 cm and flexible sigmoidoscope
upto 60 cm of colon. 239. (2)
Although diverticulosis involves mainly the left side of
235. (3) colon, bleeding occurs mostly from right side of colon.
Presence of Pseudomonas, Klebsiella and proteus is urine Right side is supplied by superior mesentric artery.
of a child of high imperforate anus is suggestive of
communication between the anus and bladder.
2014 NBE ONELINER
"Hemorrhage from colonic diverticula is the most The various levels which can be examined by various
common cause of hematochezia in patients over the age methods are:
of 60." 1) Digital examination 8 - 10 cm
2) Proctoscopy 8 - 12 cm
240. (3) 3) Rigid sigmoidoscope 18 cm
"In solitary rectal ulcer syndrome, one or more ulcers are 4) Flexible sigmoidoscope 60 cm
present in the distal rectum, usually on the anterior 5) Colonoscope 160 cm
wall." Schwartz
Causes of solitary rectal ulcer syndrome 247. (1)
o Due to a combination of It can be caused by Spirochaeta vincenti and bacillus
- internal intussusception fusiformis. Inflamed mucosa with pinpoint bleeding looks
- anterior rectal wall prolapse like a strawberry. Treatment of choice is acetarsol
- increase in intra rectal pressure suppositories with vitamin C
This combination of factors is usually due to chronic
straining as a result of constipation. 248. (2)
Symptoms pain, bleeding, mucus discharge or outlet Anorectal ring marks the junction between rectum and
obstruction anal canal.
Treatment It composes of the puborectalis, deep external sphincter,
o Nonoperative therapy (high fibre diet, defecation longitudinal muscle and highest part of intemal
training to avoid straining, and laxatives or enemas) is sphincter.
effective in the majority of patients.
o Surgery (either abdominal or perineal repair of 249. (1)
prolapse) is reserved for highly symptomatic patients, Commonest site for an anal fissure (90%) is middle
who have failed all medical intervention. posterior next most common is the anterior midline.

241. (2) 250. (4)


Digital reposition must be taught to the parents. If it fails In anal canal cancers, presently concurrent chemo
after 6 weeks of trial, 5% phenol in almond oil is injected radiotherapy is the treatment of choice. Usually a
under GA Only rarely is surgery required. combination of 5-FU and mitomycin is given.
If the lesion does not solve completely on CT & RT,
242. (2) Abdominoperineal resection is done.
Most authors claim a 2 cm margin distally to be safe in a
Ca rectum. This forms the premise in doing low and ultra
low anterior resections with the advent of staples. 251. (2)
Patients with Insulinoma do not have weight loss.
243. (4) Insulinomas are usually benign and solitary and hence
The internal sphincter is composed of longitudinal, non can be enucleated during surgery.
striated involuntary muscles supplied by autonomic
nerves. It is a continuation of the rectal muscle coat. 252. (3)
Areas of ductal dilatation altemating with areas of ductal
244. (4) stenosis are common findings in alcoholic patients who
It usually takes 6 hours for sufficient air to collect in the have severe chronic pancreatitis. This type of duct
large intestine to cast a radiological shadow. Only then obstruction cannot be relieved by sphincteroplasty
will the distance between the metal coin and air column because of multiple areas of stenosis along the duct.
be measured and 'high' anal agenesis can be Although total pancreatectomy would be a beneficial
distinguished from 'low' oral agenesis. approach, morbidity and mortality with this procedure is
extremely high. Thus the procedure of choice in such
245. (3) cases is side to side pancreaticojejunostomy
By far, the most frequent cause of a tubular inflammatory
stricture of the rectum is LGV. Frei's test is usually 253. (4)
positive. ERCP is rarely required in acute setting as management
80% of patients are women. Antibiotics are curative in is essentially conservative.
80%. 20% will require surgical excision. Serum amylase rise is nonspecific and occurs in injuries
of any intraabdominal organ.
246. (2)
254. (2)
2014 NBE ONELINER
Symptomatic annular pancreas has to be treated. 'Eradication of gall stone disease' prevents further
The treatment involves bypassing the obstruction. attacks of pancreatitis.
Duodenoduodenostomy is the most physiological
jejunostomy. 260. (2)
Ileal resection or ileal disease decreases enterohepatic
255. (3) circulation of bile salts (or acids). This decreases the
Clinical signs and symptoms of intra abdominal biliary secretion of bile salts, thus increasing the
abscesses are often minimal and delusory. However, cholesterol bile acid ratio.
recurring and persisting fever that initially is
intermittent and occurs in almost all patients, as 261. (4)
leucocytosis, shift to left. Paralytic ileus, abdominal Acalculous cholecystitis does not predispose to
distention, anorexia and vomiting also may occur. With carcinoma gall bladder. Also oral contraceptives are not
subphrenic abscess pain in upper abdomen and lower associated with carcinoma gall bladder. Porcelain gall
chest, associated with respiratory, difficulties may be bladder, Cholecystoenteric fistula, adenoma, choledochal
present. In this condition, plate like atelectasis or pleural cyst, gall stones, cholangitis and typhoid carriers are all
effusion can be seen on chest x ray above affected side of at risk for development of carcinoma gall bladder.
diaphragm in 90% of cases, and an abnormal air fluid
level can be seen in abscesses 25% of the time. Although 262. (3)
pulmonary embolism and pancreatitis are possible Triad for diagnosis of choledochal cyst is
causes of similar clinical findings, they would not account Pain abdomen
for abnormal air fluid level found in left upper quadrant Progressive Jaundice
of Lump in abdomen
patient presented in the question.
263. (1)
256. (1) ERCP will help to diagnose the tear and will be helpful
Most common tumour of pancreas is adenocarcinoma. in stent insertion.
Endocrine tumours are rare. Amongst them insulinoma is Hepaticojejunostomy is done only if there is loss of
most common. length of hepatic duct
Primary repair after exploration is a morbid procedure
257. (3) as compared to ERCP and hence will not be done.
Management of pancreatic abscess is to let the pus out by USG guided insertion of drain will also be required in
external drainage combined with antibiotics. The addition to ERCP and stenting, but is not the primary
procedure is called as Beger's procedure. treatment for CBO tear.

258. (2) 264. (4)


Cholecystectomy Cholecystectomy is not indicated for asymptomatic
Mucocele of the Gall bladder - patients with gall stones as a routine because less than
- It is one of complication of Gall stones. 25% of patients with asymptomatic gall stones will
- Caused due to obstruction of the stone at the neck of the develop symptoms that require intervention. However
bladder. one may have to do cholecystectomy for asymptomatic
- In course of time the bile is absorbed and replaced by gall stones if stone> 3 cm, patient is a diabetic, multiple
the mucus secreted by the Gall bladder epithelium. small stones, Gall bladder with polyp or calcification.
Due to this the Gall bladder may because distended and
palpable. 265. (1)
Treatment Gall stones predispose to carcinoma gall bladder, not
- The tit is early cholecystectomy cholangiocarcinoma. Other options are known
- If early tit is not done following complication can occur predisposing factors for cholangiocarcinoma, others are
- Empyema. schistosomiasis, Biliary enteric anastomosis and
- Perforation Choledochal cyst.
- Gangrene}:
266. (1)
259. (2) This patient has a recurrent CBO stone. The treatment of
Gall stone pancreatitis choice for recurrent / retained CBO stone is ERCP and
The prognosis is best in patient where pancreatitis is stone retrieval. But since the stone measures 2.5 cm it is
caused by a remediable cause such as cholelithiasis. too large to be extracted via ampulla of Vater. Hence
2014 NBE ONELINER
treatment here would be supraduodenal In thalassemia, since the anemia is due to both
choledochotomy and CBO exploration. increased destruction of red 'cells and decreased
hemoglobin production, splenectomy does not cure
267. (4) anemia as in spherocytosis, but it may reduce transfusion
Type I choledochal cyst is a fusiform dilatation and requirement.
excision is mandatory as is a premalignant condition. Splenectomy is very rarely indicated in sickle cell
Roux loop is required as end to end anastomosis is not anemia (in 3% of pts) and the most. frequent indication
possible with bile duct. is hypersplenism and acute sequestration crisis.
Although there is no benefit of splenectomy on the
268. (4) disease process.
Patients with symptomatic retained stones should
undergo endoscopic sphincterotomy after papillotomy 273. (1)
and retrieval of stones. Success rates between 85-95% The cause for recurrent symptoms is residual splenic
have been reported. It seems best suited for elderly, poor tissue, which are accessory spleens and are usually
surgical risk patients as it is least invasive. Patients in missed during splenectomy. All the mentioned options
whom endoscopic sphincterotomy and stone extraction except option 1 are common sites for accessory spleen.
has been unsuccessful, extracorporeal shock wave
lithotripsy has proven beneficial. 274. (4)
Insulinoma is equally distributed throughout the
269.(4) pancreas. Classical feature of insulinoma is fasting
Mucocele of gall bladder is nothing but sterile mucus hypoglycemia relieved by intake of glucose.
accumulation in gall bladder. Laparoscopy is GASTRINOMA is seen in the gastrinoma triangle i.e. the
contraindicated in cirrhosis due to high risk of bleeding, Triangle of passero.
in prior upper absominal surgery due to adhesions, and
in suspected Ca gall bladder for fear of incomplete 275. (1)
excision of malignancy. Transduodenal sphincteroplasty done via ERCP guided
catheter is treatment of choice for sphincter of odd
270. (3) spasm/stricture or stenosis.
A T-tube cholangiogram is taken on the 7th to 10th post-
op day to ensure there are no retained stones. If the
cholangiogram is normal, the T-tube is clamped. 276. (3)
Removed for the T-tube will depend on the material from Lockwoods' repair is low approach for the repairs of a
which it is made, as this will determine the length of time femoral hernia.
for a track to form. If it is latex rubber, the T-tube can be Lotheissein's repair is an inguinal approach to repair of
removed at 10-14 days, but if it sialistic it should be left femoral hernia through the incision over inguinal canal
for 3-4 weeks before removal. Following removed of the as for inguinal hernia.
T-tube there may be a small bile leak that persists for Stopa's repair is great prosthetic reinforcement of
1~2 days." peritoneal sac, which tackles both inguinal as well as
femoral hernia.
271. (3) Moloneys darning is done only in repairing lnguinal
Splenic artery aneurysms are commoner in females hernia.
(2:1) hence option 3 is the answer.
Usually related to main arterial trunk 277. (1)
Mostly single and symptomless. In children the omentum is small and under developed.
Occasionally may cause an audible bruit in left Hence generalised peritonitis occurs early. Ingestion of
hypochondrium and opacity in plain X ray abdomen due food, enema stimulates gut motility and hence hinders
to calcification in the sac. localisation. Option (3) causes immunosuppression.
They have to be operated as mortality is high if rupture
occurs. Procedure of choice is splenectomy and removal 278. (4)
of diseased artery. This is because contractions of the bladder commence at
apex and pass towards the base. Hence a patent urachus
272. (3) which opens into the apex of bladder in temporarily
Hereditary spherocytosis closed during micturition. This fistula will therefore
"Splenectomy is the sale treatment for hereditary present only when there is obstruction to urinary
spherocytosis and is indicated even when the anemia is outflow.
fully compensated and the patient is asymptomatic."
2014 NBE ONELINER
279. (1) tubercle. It contains perivesical fat and portion of
Bochdalek hernia is posterolateral congenital bladder. Since it occurs through a defect in abdominal
diaphragmatic hernia. Prenatal ultrasound shows wall it is a type of direct hernia.
herniated gut in thoracic cavity. It usually occurs on left
side and hence stomach and transverse colon are most 289. (4)
common contents. Progressive bacterial synergistic gangrene is due to
synergism between bacteria from the intra peritoneal
280. (4) environrnent and those colonizing the skin. The
Conservative treatment is successful in 93% of cases maximum intraperitoneal contamination among given
hence it is preferable to wait for 2 years. This is a case of option is with option 1 and 4. But in option 1 surgery
congenital umbilical hernia. Coin strapping is advised done is colostomy which affectively drains out all the
upto 2 years. If hernia persists beyond 2 years, intraperitoneal contamination. So the answer would be
herniorrhaphy is indicated. option 4.

281. (3) 290. (3)


Desmoid is a fibroma, which usually occurs in Most common presentation of tuberculous paritonitis is
musculoaponeurotic tissues of abdominal wall especially pain in abdomen (present in 90%) followed by fever
below umbilicus. It can also occur in old hernia scar and (60%), loss of weight (60%), ascites (60%)night sweats
intraperitoneally. and abdominal mass.

282. (1) 291. (3)


Although divarication of recti is seen principally in Exomphalos also mean omphalocle or umbilical hernia. It
elderly women and multipara the form that limits the is a disease of abdominal wall because it results from
divarication to above umbilicus is commoner in babies. failure of all or part of gut to return to the coelom during
early fetal life. Exomphalos has covering of amnion and
283. (4) peritoneum where as gastroschisis have no coverings.
Secondary carcinoma at umbilicus 'Sister Joseph's node'
is due to primary in stomach, ovary colon and breast. The 292. (3)
spread is believed to be along the falciform ligament Serosanguinous discharge is the forerunner of disruption
from liver mets. in 50% cases and signifies that intraperitoneal contents
are lying extraperitoneally. Pain and shock are absent.
284. (3) Signs of intestinal obstruction may be present, but is not
Age female sex and relation to menses clinch the usual. An emergency operation is required to replace
diagnosis of Endometrioma. Raspberry tumour usually bowel, relieve obstruction and resuture wound. There is
presents in childhood. biochemical evidence that healing after disruption
produces stronger wound.
285. (4)
Pregnancy and female sex are predisposing factors for 293. (2)
femoral hernia as they contribute to laxity of ligaments. Colonic perforation during colonoscopy is usually
However femoral hernia is also seen in males. It is most detected early (within 6h - of perforation) and also the
likely to strangulate. colon is already prepared before taking up patient for
colonoscopy. Hence primary closure of defect should be
286. (4) done.
Raspberry tumour is exuberant granulation tissue
pouting out from the unobliterated distal portion of 294. (1)
vitellointestinal duct. Omphalitis is infection of umbilical cord stump.
Staphylococcus aureus is the most common organism
287. (4) responsible, followed by Streptococci, E. Coli and
Familial Mediterranean fever also known as periodic Clostridium tetani.
peritonitis is commoner in females. Colchicine is not the
causative agent it is used in prevention of recurrent 295. (2)
attack of the disease. Hematoma of rectus sheath is due to tearing of inferior
epigastric artery, which is common in elderly females,
288. (1) muscular males, in multipara and in late pregnancy.
Prevesical hernia is a narrow necked hernia through a Distinguishing between rectus sheath hematoma and
gap in medial. panpfonjoint tendon just above pubic strangulated Spigelian hernia is difficult. Absence of
2014 NBE ONELINER
vomiting suggests hematoma and presence of resonance
over swelling favours Spigelian hernia.

296. (2)
Colpotomy is drainage of pelvic abcess via vaginal
fornices.

297. (4)
Infection leads to poor wound healing and subsequent
weakness of the incision site obesity is also a leading
cause, but not the most common. Persistent post
operative cough and post operative abdominal distension
are risk factors.

298. (3)
Abdominal dehiscence after an abdominal surgery is
most likely to occur between 6th and 8th post operative
days.

299. (1)
Lytle's method involves narrowing of the deep inguinal
ring so that the peritoneum does not enter thru it into the
inguinal canal. Sac enters through deep inguinal ring in
indirect hernia.

300. (1)
Obturator hernia through the obturator foramen is six
times commoner in females. Most patients are over 60
years. It is usually a Ritcher type (containing a part of
circumference of bowel within the sac). It usually
presents as a swelling in femoral triangle but a PV/PR
examination may reveal a tender lump in relation to
obturator foramen.

You might also like